71
MATEMATİK OLİMPİYATLARINA HAZIRLIK 1 TEMEL BİLGİLER -I Mustafa Özdemir İZMİR - 2016 ALTIN NOKTA YAYINEVİ

MATEMATİK OLİMPİYATLARINA HAZIRLIK 1Herbirkitaptaöncelikle,konuya veokonuileilgiliörnek ö˘gretici olabilecek sorulara yerverdim.Dahasonra,herbirkonuileilgilidünyada de˘gi¸

  • Upload
    others

  • View
    12

  • Download
    0

Embed Size (px)

Citation preview

Page 1: MATEMATİK OLİMPİYATLARINA HAZIRLIK 1Herbirkitaptaöncelikle,konuya veokonuileilgiliörnek ö˘gretici olabilecek sorulara yerverdim.Dahasonra,herbirkonuileilgilidünyada de˘gi¸

MATEMATİKOLİMPİYATLARINA

HAZIRLIK1

TEMEL BİLGİLER -I

Mustafa Özdemir

İZMİR - 2016

ALTIN NOKTA YAYINEVİ

Page 2: MATEMATİK OLİMPİYATLARINA HAZIRLIK 1Herbirkitaptaöncelikle,konuya veokonuileilgiliörnek ö˘gretici olabilecek sorulara yerverdim.Dahasonra,herbirkonuileilgilidünyada de˘gi¸

Copyright © Altın Nokta Basım Yayın Dağıtım BilişimISBN 978-975-6146-62-0

MATEMATİK OLİMPİYATLARINA HAZIRLIK-1 TEMEL BİLGİLER-I

Mustafa Ö[email protected]

Bu kitabın her hakkı saklıdır. Tüm hakları Altın Nokta Basım Yayın Dağıtım'a aittir. Kısmen deolsa alıntı yapılamaz. Metin ve sorular, kitabı yayımlayan kurumun önceden izni olmaksızın

elektronik, mekanik, fotokopi ya da herhangi bir kayıt sistemiyle çoğaltılamaz, yayımlanamaz.

Bu kitaptaki TÜBİTAK Matematik Olimpiyat Soruları, TÜBİTAK – Bilim İnsanı Destekleme DaireBaşkanlığından izin alınarak yayımlanmaktadır.

Genel Yayın YönetmeniHalil İ. AKÇETİN

Kapak-DizgiAltın Nokta Dizgi-Grafik

BaskıERTEM BASIM YAYIN DAĞ. SAN. TİC.LTD. ŞTİ.

Nasuh Akar Mah. 25. Sok. No: 19 Çankaya / ANKARATel: 0 (312) 640 16 23

Yayın - DağıtımAltın Nokta Basım Yayın Dağıtım859 Sk. No:1/Z-4 Konak / İZMİR

Tel- Faks : 0 (232) 441 25 95

www.nokta2000.com www.altinnokta.com.tr

www.kitapana.com

[email protected]@altinnokta.com.tr

[email protected]

Ocak - 20165. Basım

Page 3: MATEMATİK OLİMPİYATLARINA HAZIRLIK 1Herbirkitaptaöncelikle,konuya veokonuileilgiliörnek ö˘gretici olabilecek sorulara yerverdim.Dahasonra,herbirkonuileilgilidünyada de˘gi¸

Önsöz

Antalya Matematik Olimpiyatlarına ögrenci hazırlayan bazı ögretmenlerimizinTürkçe kaynak bulamamaktan sikayet etmeleri üzerine yazmaya karar verdigim vebir kitaptan olusacagını düsünerek basladıgım olimpiyatlara hazırlık kitabı, bir andabes ciltlik güzel bir kaynak oldu. Bu kitaplarda, hemen hemen her konuya ve bukonularla ilgili degisik sorulara yer vermeye çalıstım.

Aslında, Ingilizce bilen her ögrenci internette birçok olimpiyat kaynagına ulasa­bilir. Fakat, konularına göre yazılmıs ve konularına göre sorular verilmis MatematikOlimpiyatlarına Hazırlık Kitapları, olimpiyatlara yeni baslayan ögrenciler ve olim­piyatlara ögrenci hazırlayan ögretmenler için büyük bir kolaylık saglamıstır. Bu ki­taplardan ögrenilen temel bilgiler sayesinde, farklı dillerden ve farklı kaynaklardanulasılan bir çok olimpiyat sorusunun ve çözümünün daha çabuk anlasılır hale gele­cegini umuyorum.

Kitaplarla ilgili Türkiye’nin hemen hemen her yerinden hem ögrencilerden hemde ögretmenlerimizden tesekkür e­postaları ve telefonları almaktayım. Türkiye’deböyle bir kaynagın olmasından dolayı tesekkür eden birçok ögrenci ve ögretmeni­miz, konularına göre soruların verilmis olmasının, her konunun detaylı sekilde in­celenmis olmasının ve anlasılır bir dille yazılmıs olmasının, kendilerine büyük ko­laylık sagladıgını e­postalarında ifade ediyorlar. Kitapta bulunan bazı hataları da banabildiren ögrenci ve ögretmenlerimiz sayesinde, daha sonraki basımlarda, bu hatalarıngiderilmesine çaba sarf ediyorum. Her ne kadar tashihini yapmıs olsam da, sorularınzorlugu ve çoklugu ve zamanın darlıgı bazen hataları görmemize engel oluyor. Ki­taplardaki olabilecek diger hatalarımı ve kitapla ilgili görüs ve düsüncelerinizi, [email protected] e­posta adresine gönderirseniz sevinirim.

Bu kitaplarla güzel ülkeme çok küçük de olsa, bir faydam olmussa, bu benimiçin büyük bir mutluluktur.Hangi Ciltte Hangi Konular Var?

Birinci ve ikinci ciltte, olimpiyatlar için en gerekli temel kavramların ve yön­temlerin verilmesi amaçlandı. Bunun için, temel kavramlar, tanımlar, gösterimlerverilerek, problem tipleri, çarpanlara ayırma, çözümleme, toplamlar, kombinatorik,binom açılımı, ispat yöntemleri konuları ele alındı.

Üçüncü ciltte ise, sayılar teorisi konusu ele alınarak, bölünebilme, asal sayılar,obeb­okek, modüler aritmetik, Fermat, Euler, Wilson teoremleri, Çin kalan teoremi,denklikler, tamdeger, konuları verildi.

Dördüncü ciltte ise, fonksiyonlar, polinomlar, polinom denklemler ve esitsizlik­ler, diziler, denklemler ve denklem sistemleri konularına yer verildi.

Besinci cillte ise, logaritma ve trigonometri bilgisi, limit, süreklilik, türev, fonksi­yonel denklemler ve esitsizlikler konuları verildi.

Page 4: MATEMATİK OLİMPİYATLARINA HAZIRLIK 1Herbirkitaptaöncelikle,konuya veokonuileilgiliörnek ö˘gretici olabilecek sorulara yerverdim.Dahasonra,herbirkonuileilgilidünyada de˘gi¸

Her bir kitapta öncelikle, konuya ve o konu ile ilgili örnek ögretici olabileceksorulara yer verdim. Daha sonra, her bir konu ile ilgili dünyada degisik olimpiyat-larda sorulmus soruları ve çözümlerini ilave ederek, ögrencilerin karsılasabilecegifarklı soruları görmesini sagladım.Türkiye’deki Matematik Olimpiyatları Konusunda Kısa Bilgi

Türkiye’de olimpiyat etkinlikleri, TÜBITAK Bilim Insanı Destekleme DaireBaskanlıgı (BIDEB) tarafından yürütülmektedir. Bu çalısmalar hem ulusal düzeydehem de uluslararası düzeyde yapılmaktadır. Ulusal düzeyde gerçeklestirilen Ilkögre­tim Matematik Olimpiyatı ile Liseler Için Matematik Olimpiyatları sonuçlarına göreülkemizi Uluslararası yarısmalarda temsil edecek takımlar belirlenmektedir. Ulus­lararası Bilim Olimpiyatlarında ülkemizi temsil edecek takımlar matematik olimpiyatkamplarında basarılı olmus ögrencilerin, çesitli sınavlar sonucunda seçilmeleriyle olus­maktadır. Su ana kadar katıldıgımız Uluslararası Matematik Olimpiyatlarında,Umut Varolgünes, Melih Üçer, Ömer Faruk Tekin, Cafer Tayyar Yıldırım, Selim Ba­hadır (2 kez), Nizameddin Ordulu, Mehmet Bumin Yenmez ülkemize altın madalyakazandıran ögrencilerdir.

Son yıllarda, birçok üniversite lise ögrencilerine yönelik olarak matematik olim­piyatları düzenlemektedir. Bunlardan en eskisi Akdeniz Üniversitesi tarafından düzen­lenen Ulusal Antalya Matematik Olimpiyatlarıdır, bu olimpiyat birincisi test ve ikin­cisi klasik olarak iki asamada yapılmaktadır. Yine, Fatih, Koç, Dogus, Mersin, Sa­bancı üniversiteleri de matematik olimpiyatı düzenleyen üniversitelerden bazılarıdır.

Matematik Olimpiyatlarına Hazırlanan Bir Ögrenci Ne Kazanır?

Matematik olimpiyatlarına hazırlanmak hem zor hem de zevklidir. Matematikolimpiyatlarına hazırlanan bir ögrenci sınavın sonucunda hangi dereceyi alırsa alsınasla kaybetmez. Ögrendigi konular ve zor soruların yanında, beynini zorlamasıufkuna açmasına ve ileride zor problemler ile karsılastıgında daha saglıklı ve dahatutarlı yorumlar yapmasını saglayacaktır. Sporla ugrasan bir sporcu katıldıgı olim­piyatta basarılı olamasa bile, hazırlanma asamasında vücudunun saglıklı olması içinyaptıgı çalısmaların faydasını gördügü gibi, matematik olimpiyatlarına hazırlanan birögrenci de, zor problemlere kafa yormasının sonucu olarak beynini gelistirir. Insan­lar düsündükçe aklını kullandıkça , matematik problemi çözdükçe beyin hücrelerininyolları açılır. Bilim adamları, normal insanların mevcut beyin kapasitelerinin çokaz bir kısmını kullanabildigini söylemektedirler. Bu kapasite elbette sıradan islerleugrasarak, beyni yormayarak, basit ve birbirine benzeyen problemleri çözerek artma­yacaktır. Beyni yormak gerekir. Beyni zorlamak, sürekli yeni problemlerle mesguletmek gerekir. Beyin hücreleri kullanılmaz ise kaybedilir. O halde, bir matematikyarısmasına girsek de girmesek de zor sorular ile ugrasmalıyız.

Page 5: MATEMATİK OLİMPİYATLARINA HAZIRLIK 1Herbirkitaptaöncelikle,konuya veokonuileilgiliörnek ö˘gretici olabilecek sorulara yerverdim.Dahasonra,herbirkonuileilgilidünyada de˘gi¸

Matematik Olimpiyatlarına Nasıl Hazırlanılmalı?

Matematik Olimpiyatlarına hazırlanmak gerçekten zordur. Zaman ister. Tıpkıolimpiyata hazırlanan bir haltercinin sürekli kendini gelistirmesi, yavas yavas agır­lıkları kaldırması ve bunu basarabilmek içinde gerekli zamanı harcayıp vücudunugelistirmesi gibi, yavas yavas ilerlenmesi gereken bir çalısmadır. Olimpiyat sorularınıçözmeye yeni baslayan birisine, bazı soruların oldukça zor gelmesi normaldir. Bu bi­raz bilgiye, biraz tecrübeye biraz da püf noktalı sorulara hazırlıklı olmaya göre degisir.Soruların zorluk derecesi, elbetteki, bir halterin agırlıgı gibi net olarak ifade edilmesede, bildiginiz bir konuda sorulan bir sorudaki ince bir püf nokta o soruyu çok zor halegetirebilir. Bir soru ögrenildikten sonra kolaydır. Ögreninceye kadar zor bir sorudur.Bu kitabın amaçlarından biri de size göre zor olan soruların sayısının azalmasınayardımcı olmaktır. Olimpiyatlara hazırlanan bir ögrenci herseyden önce, kararlıolmalı, kendine güvenmeli, fakat ne kadar kendine güvenirse güvensin yapamaya­cagı soruların oldugunun farkında olup, çözemedigi sorular karsısında umutsuzlugadüsmek yerine, çözemedigi soruların çözümlerini ögrenerek ilerlemesi gerektigininbilincinde olmalıdır. Kısaca, matematik olimpiyatlarına hazırlık, kararlılık, sabır veazim isteyen bir istir. Acele etmemek gerekir. Hatta bazı soruların çözümü de an­lasılamayabilir veya bir sorunun çözümü ögrenildikten sonra tekrar karsılasıldıgındao soruyu yapamayabilirsiniz. Ögrencilerden, bu konu ile ilgili en çok karsılastıgımsoru, "çözümünü gördügümüz zaman anlıyoruz ama kendimiz yapamıyoruz, ne yap­malıyız?" sorusudur. Aslında bu normaldir. Olimpiyat sorularının kendine has çözmeyöntemleri olabilir. Bu yöntemleri bir anda ögrenmek elbette kolay degildir. Bukitapta konular ve konu ile ilgili sorulan sorular mümkün oldugu kadar, o konuyagelinceye dek ögrenilen bilgileri içerecek sekilde ele alınmıstır. Bir soruyu çözerken,soruyu önce kendiniz çözmeye çalısınız. Çözemez iseniz, çözümünü inceleyipnasıl bir yöntem kullanıldıgını inceleyiniz ve soruda püf nokta var ise, o püf nok-tayı mutlaka görmeden soruyu geçmeyiniz. Sorunun çözümünü anlamaz iseniz, bukonu ile ilgili bilgilerinizin eksik olabilecegini göz önünde bulundurarak umutsuzlugakapılmayınız. Unutmayın sizi zorlayan her soru sizin için zor ve güzel bir sorudur.Bazı sorularda hata da olabilir. Bu tür hataları bildirirseniz, kitabın bundan sonrakibasımlarında daha hatasız olarak size ulastırabiliriz.

Page 6: MATEMATİK OLİMPİYATLARINA HAZIRLIK 1Herbirkitaptaöncelikle,konuya veokonuileilgiliörnek ö˘gretici olabilecek sorulara yerverdim.Dahasonra,herbirkonuileilgilidünyada de˘gi¸

Kısa Özgeçmis

Mustafa Özdemir, 1975 yılında Konya’nın Bozkır ilçesinde dogdu. Ilk, orta ve liseögrenimini Antalya’da tamamladı. 1992 yılında girdigi Dokuz Eylül ÜniversitesiBuca Egitim Fakültesi Matematik Ögretmenligi Bölümü’nden 1996 yılında mezunoldu. 1999 ­ 2007 yılları arasında, Akdeniz Üniversitesi, Fen Bilimleri Enstitüsü,Matematik Anabilim Dalında yüksek lisans ve doktorasını tamamladı. Halen, Akde­niz Üniversitesi Matematik Bölümünde çalısmaktadır.

Besinci Baskı Için Tesekkür.

Kitabın daha önceki baskılarında hatalı soru çözümleri, baskı hataları, eksikçözümler, yanlıs ifade edilisler ile ilgili birçok hatalarımı görüp bildiren, basta Bahçe­sehir Üniversitesi Ögretim Üyesi Yrd. Doç. Dr. Maksat Ashyraliyev olmak üzere,Baser Kandehir’e, Resit Kaya’ya, Mahmut Bektas’a, Taha Eyüp Korkmaz’a,Salih Can’a, Ahmet Arduç’a, Ebubekir Celayir’e, Oguzhan Yılmaz’a, AhmetAlaydın’a ve Mustafa Küçük’e çok tesekkür ediyorum. Onların da katkılarıyla kitapçok daha hatasız hale gelmistir.

Ayrıca, Matematik Olimpiyatlarına Hazırlık kitaplarını yazmamda bana destekolan degerli hocalarım, Prof. Dr. Abdullah Aziz Ergin’e, Prof. Dr. Halil IbrahimKarakas’a, Prof. Dr. Ilham Aliyev’e, Prof. Dr. Hasan Ali Çelik’e ve Prof. Dr. AliNesin’e ve esim Burcu Özdemir’e tesekkür ederim.

Son olarak, Altın Nokta Yayınevi olarak, kitabı yayınları arasına alarak basımı vedagıtımı konusunda her türlü fedakarlıgı yapan Halil Ibrahim Akçetin’e ve degerliesi Leyla Akçetin’e çok tesekkür ederim.

Mustafa ÖzdemirAntalya ­ 2015

[email protected]

Hayatta iken degeri yeterince bilinmeyen tüm anneler adınaAnnem Hayriye Özdemir’e

Page 7: MATEMATİK OLİMPİYATLARINA HAZIRLIK 1Herbirkitaptaöncelikle,konuya veokonuileilgiliörnek ö˘gretici olabilecek sorulara yerverdim.Dahasonra,herbirkonuileilgilidünyada de˘gi¸

Içindekiler

BIRINCI BÖLÜM

Temel Bilgiler

Sayılar 11Ardısık Sayıların Toplamı 16Bölünebilme Kuralları 19Bir Sayının Pozitif Bölenlerinin Sayısı, Toplamı ve Çarpımı 25EBOB ­ EKOK 29Modüler Aritmetik 34Basit Denklem Çözümleri ve Sayıların Özelliklerinin Kullanılması 38Basit Esitsizlikler 44Faktöriyel Kavramı 46Bir Sayının Tam Kısmı 50Mutlak Deger 54Üslü ve Köklü Sayılar 58Oran ­ Orantı 64Karısık Örnekler 65Çözümlü Test 1 73Çözümler 78TÜBITAK SORULARI (Temel Bilgiler) 87TÜBITAK SORULARININ ÇÖZÜMLERI (Temel Bilgiler) 99ULUSAL ANTALYA MATEMATIK OLIMPIYATI SORULARI 119

IKINCI BÖLÜM

Problemler

Yas Problemleri 125Isçi ­ Havuz Problemleri 127Hareket Problemleri 131Yüzde ­ Faiz Problemleri 133Karısım Problemleri 136Saat Problemleri 137Sınav Problemleri 138Tahtadaki Sayı Problemleri 139

Page 8: MATEMATİK OLİMPİYATLARINA HAZIRLIK 1Herbirkitaptaöncelikle,konuya veokonuileilgiliörnek ö˘gretici olabilecek sorulara yerverdim.Dahasonra,herbirkonuileilgilidünyada de˘gi¸

Tartı Problemleri 143Sayı Tablosu ve Sihirli Kare Problemleri 145Mantık Problemleri 147

Oyun ve Turnuva Problemleri 148Çember Etrafına Sayı Yerlestirme Problemleri 151Çözümlü Test 2 155Çözümler 160TÜBITAK SORULARI (Problemler) 167TÜBITAK SORULARININ ÇÖZÜMLERI (Problemler) 181ULUSAL ANTALYA MATEMATIK OLIMPIYATI SORULARI 203

ÜÇÜNCÜ BÖLÜM

Çarpanlara Ayırma ve Özdeslikler

Çarpanlara Ayırma Yöntemleri 207Özdeslikler 214Karısık Örnekler 230Çözümlü Test 3 239Çözümler 244TÜBITAK SORULARI (ÇarpanlaraAyırma) 255TÜBITAK SORULARININ ÇÖZÜMLERI (Çarpanlara Ayırma) 261ULUSAL ANTALYA MATEMATIK OLIMPIYATI SORULARI 273

DÖRDÜNCÜ BÖLÜM

Çözümleme ve Taban Aritmetigi

Çözümleme 275Rakam Degistirme veya Silme 281Karısık Örnekler 284Çözümlü Test 4 295Çözümler 302TÜBITAK SORULARI (Çözümleme) 317TÜBITAK SORULARININ ÇÖZÜMLERI (Çözümleme) 323ULUSAL ANTALYA MATEMATIK OLIMPIYATI SORULARI 335

BESINCI BÖLÜM

Esitsizliklere Giris

Aritmetik ­ Geometrik ­ Harmonik Ortalama ve Esitsizlikleri 337

Page 9: MATEMATİK OLİMPİYATLARINA HAZIRLIK 1Herbirkitaptaöncelikle,konuya veokonuileilgiliörnek ö˘gretici olabilecek sorulara yerverdim.Dahasonra,herbirkonuileilgilidünyada de˘gi¸

Cauchy ­ Schwartz Esitsizligi 346Karısık Örnekler 350Çözümlü Test 5 352Çözümler 355ULUSAL ANTALYA MATEMATIK OLIMPIYATI SORULARI 360YANIT ANAHTARI 364

Page 10: MATEMATİK OLİMPİYATLARINA HAZIRLIK 1Herbirkitaptaöncelikle,konuya veokonuileilgiliörnek ö˘gretici olabilecek sorulara yerverdim.Dahasonra,herbirkonuileilgilidünyada de˘gi¸
Page 11: MATEMATİK OLİMPİYATLARINA HAZIRLIK 1Herbirkitaptaöncelikle,konuya veokonuileilgiliörnek ö˘gretici olabilecek sorulara yerverdim.Dahasonra,herbirkonuileilgilidünyada de˘gi¸

Temel Bilgiler

1.1 Sayılar

I N = 0 1 2 3 kümesinin her bir elemanına dogal sayı denir.Z = −2−1 0 1 2 kümesinin her bir elemanına tamsayı denir.Z+ = 1 2 3 kümesine pozitif tamsayılar kümesi veZ− = −3−2−1 kümesine de negatif tamsayılar kümesi denir.Sıfır bir tamsayıdır, fakat pozitif veya negatif degildir.

I ve tamsayı ve 6= 0 olmak üzere,

seklinde yazılabilen sayılara rasyonel

sayı denir. Rasyonel sayılar kümesini,

Q =n: ∈ Z ve 6= 0

oile gösteririz. Virgülden sonraki bir kısımı periyodik olarak devam eden rasyonelsayılara devirli ondalık sayılar denilir. Devreden kısım üstü çizgili olarak gösterilir.Devirli bir ondalık sayıyı rasyonel olarak ifade edebiliriz. Bunun için, verilen devirlisayı, devreden kısım sadece bir kez yazılmak sartıyla, virgülsüz kabul edilir. Sonra, busayıdan, sürekli tekrar eden devirli kısım haricindeki sayı çıkarılır. Elde edilen sayı,virgülden sonraki devreden kadar 9 devretmeyen kadar 0’ın yanyana yazılmasıyla eldeedilen sayıya bölünür. Örnegin, devirli sayısı, −

9990olarak yazılır.

I Rasyonel olmayan sayılara irrasyonel sayılar denir. Yani,

seklinde yazı­

lamayan, virgülden sonrası düzenli bir biçimde tekrarlanmayan sayılara irrasyonelsayılar denir. Örnegin,

√2,√3 ve sayılarının kesirli olmadıkları bilinir. Ilk ikisinin

kanıtı kolaysa da üçüncüsünün kanıtı zordur. Irrasyonel sayıları, Q0 ile gösterecegiz.I Rasyonel sayılar kümesiyle irrasyonel sayıların kümesinin birlesimine reel (gerçel)sayılar kümesi denir. R = Q ∪Q0 ile gösterilir.I Bir rasyonel sayıyla bir irrasyonel sayının toplamı irrasyoneldir. Bunu göstere­lim.

∈ Q ve ∈ Q0 olsun. + = ∈ Q olsaydı, = − esitligine göre, ikirasyonel sayının farkı yine bir rasyonel sayı olacagından, sayısı da rasyonel olurdu.Fakat, sayısını bir irrasyonel sayı kabul etmistik. O halde, bir rasyonel sayıyla birirrasyonel sayının toplamı rasyonel olamaz.I Iki irrasyonel sayının toplamı irrasyonel sayı olmayabilir. Örnegin,

√2 ve 1−√2

sayıları irrasyonel sayılar olmasına ragmen toplamları 1’dir ve rasyoneldir.I Iki irrasyonel sayının çarpımı irrasyonel sayı olmayabilir. Örnegin,

√3 ve

√3

irrasyonel sayılarının çarpımı 3’dür ve rasyoneldir.I Bir rasyonel sayı ile bir irrasyonel sayının çarpımı irrasyoneldir denilemez. Ras­yonel sayılardan birini, 0 alırsak, sonuç 0, yani bir rasyonel sayı olur.

Page 12: MATEMATİK OLİMPİYATLARINA HAZIRLIK 1Herbirkitaptaöncelikle,konuya veokonuileilgiliörnek ö˘gretici olabilecek sorulara yerverdim.Dahasonra,herbirkonuileilgilidünyada de˘gi¸

12 Matematik Olimpiyatlarına Hazırlık 1

I [ ] gösterimi ve dahil ile arasındaki reel sayıları gösterir. Yani, ∈ [ ]ise, ≤ ≤ ’dir. ( ] gösterimi, hariç ve dahil ile arasındaki reel sayılarıgösterir. Yani, ∈ ( ] ise, ≤ olur ve eger ise, ∈ ( )’dir.I bir tamsayı olmak üzere, 2 seklinde ifade edilen sayılara çift sayı, 2 − 1seklinde ifade edilen sayılara da tek sayı denir. Çift sayıları, Ç ile, tek sayıları da ile gösterecegiz.

± = Ç, ± Ç = Ç ± Ç = Ç, · = · Ç = Ç, Ç · Ç = Ç,

ve ∈ Z+ için = , Ç = Ç

biçimindedir.I 1 ve kendisinden baska pozitif tam böleni olmayan, 1’den büyük tamsayılara asalsayılar denir. 1 asal sayı degildir. 2’den baska çift asal sayı yoktur.

Bir sayısının asal olup olmadıgını nasıl anlamak için ’yi√’den küçük asal

sayılara bölerek kontrol edebiliriz. Çünkü = ve ≤ ise ≥ ’dir. Dolayısıyla asal degilse

√’den küçük bir asal sayıya bölünür.

Örnegin, 199 sayısı asaldır. Çünkü,√199 ' 142 oldugundan 14’ten küçük 2

3 5 7 11 ve 13 sayılarının hiçbiri 199 sayısını bölmez.

Örnek 1 3n− 10, 6n− 13 ve 5n− 13 sayılarının üçü de asal sayı olacak se­kilde kaç pozitif tamsayısı vardır?Çözüm : Verilen üç sayının toplamı,

(3− 10) + (5− 13) + (6− 13) = 14− 36oldugundan çifttir. Üç asal sayının toplamının çift olabilmesi için birinin mutlaka 2olması gerekir. Buna göre,

3−10 = 2 ise, = 4 olacagından, 5−13 = 7 ve 6−13 = 11 oldugundanüçü de asal olur.

5− 13 = 2 ise, = 3 olur, fakat, 3− 10 asal olmaz.6− 13 = 2 olması ise mümkün degildir.

O halde, sadece = 4 degeri için üç sayıda asal sayıdır.

Örnek 2 p, p+ 10 ve p+ 14 sayıları asal olacak sekilde kaç p sayısı vardır?Çözüm : Bu sayıların 3’e bölümünden kalanlar sırasıyla, ’nin, +1’in ve +2’nin3’e bölümünden kalanlara esittir ve bu kalanlar birbirinden farklıdır. Bu durumda, busayılardan biri 3’e bölünmelidir. Sayılar asal oldugundan, en küçük olan sayısının 3olması gerekir. Böylece diger sayılar 13 ve 17 olur.

Page 13: MATEMATİK OLİMPİYATLARINA HAZIRLIK 1Herbirkitaptaöncelikle,konuya veokonuileilgiliörnek ö˘gretici olabilecek sorulara yerverdim.Dahasonra,herbirkonuileilgilidünyada de˘gi¸

Temel Bilgiler 13

Örnek 3 100 tane pozitif tamsayının toplamı çarpımından büyüktür. Bu sayılar­dan en fazla kaçı 1’den büyük olabilir?Çözüm : 1’den farklı pozitif sayıların en çok olması için, 1’den büyük en küçüktamsayı olan 2 sayısını kullanmalıyız. Daha büyük sayıları kullanmamız çarpımındaha hızlı büyümesine neden olur. Buna göre, 7 tane 2 kullanmıs olsak, geriye 93tane 1 kalır ki, bunların çarpımı 27 = 128 iken toplamları, 93 + 7 · 2 = 107 olur vekosul saglanmaz. Bu nedenle, 2 sayısı 7’den küçük olmalıdır. 6 tane 2 olursa, çarpım26 = 64 ve toplam ise 94 + 6 · 2 = 106 olur. Yani, toplam çarpımdan büyük olur. Ohalde, 1’den büyük olan sayıların sayısı en fazla 6 olabilir.

Alıstırma : 100 tane pozitif tek sayının toplamı çarpımından büyüktür. Bu sayılar­dan en fazla kaçı 1’den büyük olabilir?

Örnek 4105

4,110

5,115

6,120

7,125

8, ... rasyonel sayılarından kaçı tamsayıdır?

Çözüm : Verilen rasyonel sayıların payı 5’er 5’er artmakta, paydası ise 1’er 1’erartmaktadır. Dikkat edilirse sayıların herbiri, = 1 2 3 olmak üzere,

100 + 5

+ 3

formundadır ve bu sekilde devam etmektedir. Bu kesiri düzenlersek,100 + 5

+ 3=

5 (+ 3) + 85

+ 3=5 (+ 3)

+ 3+

85

+ 3

= 5 +85

+ 3= 5 +

5 · 17+ 3

elde edilir. Bu ifadenin bir tamsayı olabilmesi için, + 3 sayısı 85’e bölünmelidir.Buna göre,

+ 3 ∈ 5 17 85olabilir. Yani, = 2 = 14 ve = 82 için tamsayı elde ederiz.

Örnek 5 n2+n3 sayısı tamkare olacak sekilde 100’den küçük kaç n pozitif tam­sayısı vardır?Çözüm : 2 + 3 = 2 (+ 1) sayısının tamkare olabilmesi için, + 1 sayısı birtamkare olmalıdır. Yani, ∈ Z+ için,

= 2 − 1 ve ≤ 100olmalıdır. O halde, ∈ 2 3 10 için 2 + 3 sayısı tamkare olur ve 9 tanedir.

Page 14: MATEMATİK OLİMPİYATLARINA HAZIRLIK 1Herbirkitaptaöncelikle,konuya veokonuileilgiliörnek ö˘gretici olabilecek sorulara yerverdim.Dahasonra,herbirkonuileilgilidünyada de˘gi¸

14 Matematik Olimpiyatlarına Hazırlık 1

Örnek 6 a, b, c, d ve e birbirinden farklı birer rakam ve " : " isareti bölmeislemini göstermek üzere, a : b : c : d : e isleminde parantezler kullanılarak eldeedilebilecek en büyük sayı kaçtır?Çözüm : Herhangi, : : : · · · : ifadesinde, ve hariç diger tüm sayılarıparantezler kullanarak pay veya paydadan istedigimiz yere koymamız mümkündür. kesinlikle kesirin “payın”da, ise “payda”sındadır. Buna göre, : : : : ifadesinin, en büyük olması için,

= 9 = 8 = 7, = 6 ve = 1

alınabilir. Bu durumda sayımız, en büyük9 · 8 · 7 · 6

1= 3024

olabilir ve parantezlerle 9 [((18) 7)] 6 seklinde gösterebiliriz.

Örnek 7 10’dan küçük olan ve en sadelesmis durumda paydası 30 olan tüm pozi­tif rasyonel sayıların toplamını bulunuz. (AIME 1992)

Çözüm : 0 ile 1 arasında istenen sekildeki rasyonel sayılar1

307

3011

3013

3017

3019

3023

3029

30

sayılarıdır. Bu sayıların toplamı ise1 + 7 + 11 + 13 + 17 + 19 + 23 + 29

30= 4

olur. 1 ve 2 arasındaki istenen sekildeki sayıları, bu sayılara 1 ekleyerek elde ede­biliriz. 2 ve 3 arasındaki istenen sayılar için, bu sayılara 2 ekleyerek elde edilebilir.Benzer sekilde düsünerek, istenen toplam

10 · 4 + 8 (1 + 2 + · · ·+ 9) = 40 + 8 · 9 · 102

= 400

bulunur.

Örnek 8 Rakamları birbirinden farklı 9 basamaklı bir sayının herhangi yedi raka­mı silindiginde elde edilen iki basamaklı sayıya özsayı diyelim. Özsayıların sadecebirinin asal olabilmesi için, 9 basamaklı sayıda hangi rakam kullanılmamalıdır?Çözüm : 0, 5 ve çift rakamları sayının en sonunda kullanalım. Bu sayılar özsayınınsonunda kaldıgında özsayı asal olamaz. Örnegin, ...246850 yazabiliriz. Geriye 1,3, 7 ve 9 rakamları kaldı. 7 rakamını 3, 1, 9 ile kullandıgımızda daima asal sayı eldeedecegimizden 7 sayısını kullanmamalıyız. 91 ve 93 sayıları da asal olmadıgından,

913246850

sayısı istenen sekilde sayıdır ve burada sadece 13 sayısı asal olabilir.

Page 15: MATEMATİK OLİMPİYATLARINA HAZIRLIK 1Herbirkitaptaöncelikle,konuya veokonuileilgiliörnek ö˘gretici olabilecek sorulara yerverdim.Dahasonra,herbirkonuileilgilidünyada de˘gi¸

Temel Bilgiler 15

Örnek 9 a, b ve c birbirinden farklı rakamları göstermek üzere,1

n= 0, abc

olacak sekilde kaç n pozitif tamsayısı vardır?

Çözüm :1

= 0 esitliginden,

1

=

999veya =

999

=33 · 37

olmalıdır. Buna göre, üç farklı rakamdan olusacak sekilde, 33 · 37 sayısınınçarpanlarını incelersek, istenen sekildeki sayılar sadece 037 ya da 027 olabilir. (Digerçarpanlar üç farklı rakamı içermezler. 1, 3, 9, 111, 333, 999.) O halde,

=999

37= 27 ve =

999

27= 37

oldugundan, verilen esitligi saglayan iki degisik sayısı vardır.

Örnek 10 Toplamları 500 olan pozitif tamsayıların çarpımı en büyük kaç ola­bilir?Çözüm : En büyük çarpanı aradıgımız için, toplanan sayılar 4’ten büyük olamaz.Çünkü, 4’ten büyük herhangi bir sayısı için, 3 (− 3) çarpımı daha büyüktür.Örnegin, 5 yerine, 3 + 2 yazılırsa, 3 · 2 = 6 oldugundan, daha büyük bir çarpımelde etmemizi saglar. Toplananlar arasında 1 sayısı da olamaz. Çarpımı degistirmez.Toplanan sayılar arasında 4 yerine, 2 tane 2 kullanabiliriz. Bu durumda çarpım degis­meyecektir.

Sonuç olarak, toplamları sabit olan sayıların çarpımının en büyük olması için,toplanan sayıları 3 ve 2 sayılarından olusturmak gerekir. Fakat, 2’den fazla 2 kullan­mak yerine, 3 kullanmak daha büyük sayıya ulasmamızı saglayacaktır. Bunu bir kaçörnekle görmek mümkündür. Örnegin, 6 = 2 + 2 + 2 için, çarpım 8 elde edilirken,6 = 3 + 3 için çarpım 9 elde edilir.

O halde, toplananlar arasında en fazla 2 tane 2 olmalıdır. 500 = 166 · 3 + 2oldugundan, toplanan sayıların 166 tanesi 3 ve 1 tanesi 2 olmalıdır. Bu durumda enbüyük çarpım : 31662 olur.Soru : Siz de, toplamları 301 olan sayıların çarpımlarının en büyük 4 · 399 olabile­cegini görünüz.

Page 16: MATEMATİK OLİMPİYATLARINA HAZIRLIK 1Herbirkitaptaöncelikle,konuya veokonuileilgiliörnek ö˘gretici olabilecek sorulara yerverdim.Dahasonra,herbirkonuileilgilidünyada de˘gi¸

16 Matematik Olimpiyatlarına Hazırlık 1

1.2 Ardısık Sayıların Toplamı

I Bir sayı dizisinin ardısık terimlerine, dizinin ardısık sayıları denir. bir tamsayıolmak üzere, ardısık üç tamsayıyı +1 +2 ve ardısık üç çift sayıyı da 2 2+22+4 seklinde yazarız. En genel halde, bir tamsayısının katı olan ardısık üç tamsayısırasıyla; , + ve + 2 biçiminde yazılır. Hep aynı deger artarak (veyaazalarak) elde edilen ardısık sayılara, aritmetik olarak artan (veya azalan) sayılardenir.Her defasında artan, ’den baslayıp ’de biten ardısık sayıların sayısı,

+ 1

formülüyle bulunur. Örnegin, 5 12 19 26 96 sayı dizisinde96− 57

+ 1 = 14

sayı vardır. Ardısık sayıların toplamı da,

+ (+ ) + (+ 2) + · · ·+ =+

2·µ−

+ 1

¶ile bulunur. Yani terim sayısı, ilk ve son terimin ortalamasıyla çarpılır. Örnegin

5 + 12 + 19 + 26 + · · ·+ 96 = 14 · 96 + 52

= 707

bulunur. Buna göre, ardısık ilk sayının toplamı,

1 + 2 + 3 + · · ·+ = (+ 1)

2

ile ve ardısık ilk tek sayının toplamı da1 + 3 + 5 + · · ·+ (2− 1) = 2

formülüyle hesaplanabilir. Örnegin,10 + 11 + 12 + · · ·+ 20 = (1 + 2 + 3 + · · ·+ 20)− (1 + 2 + · · ·+ 9)

=20 · 212− 9 · 10

2= 165

seklinde hesaplanabilir.

Örnek 11 1’den 1071’e kadar olan tüm tamsayıları bir sırada12345678910111210701071

seklinde yazarak yeni bir sayı elde ediliyor.a) Bu sayı kaç basamaklıdır?b) Bu sayının 1000’inci rakamı kaçtır?c) Bu sayının rakamları toplamı kaçtır?

Page 17: MATEMATİK OLİMPİYATLARINA HAZIRLIK 1Herbirkitaptaöncelikle,konuya veokonuileilgiliörnek ö˘gretici olabilecek sorulara yerverdim.Dahasonra,herbirkonuileilgilidünyada de˘gi¸

Temel Bilgiler 17

Çözüm : a) Basamak sayısına göre kaçar rakam kullanıldıgını hesaplayalım.Bir basamaklı 1 2 9 sayılarında 9 rakam kullanılır.Iki basamaklı 10 11 99 sayılarının sayısı : 99− 10 + 1 = 90’dır ve 90 · 2 = 180rakam kullanılır.Üç basamaklı 100 101 102 999 sayılarının sayısı : 999− 100 + 1 = 900’dür ve900 · 3 = 2700 rakam kullanılmıstır.Dört basamaklı 1000 1001 1071 sayılarının sayısı da 1071 − 1000 + 1 = 72’dirve 72 · 4 = 288 rakam kullanılır. Böylece, toplam 9 + 180 + 2700 + 288 = 3177

rakam kullanılır. Yani, sayı 3177 basamaklıdır.

b) 9 tane bir basamaklı sayı ve 90 tane de iki basamaklı sayı vardır. O halde, bir veiki basamaklı sayıların kullanılmasıyla

9 + 90 · 2 = 189rakam kullanılır. O halde, 1000−189 = 811 rakam üç basamaklı sayılardan olacaktır.

811 = 3 · 270 + 1oldugundan üç basamaklı 270 sayı kullanılmıs ve 271’inci sayının ilk rakamı bizeistedigimiz sayıyı verecektir. Üç basamaklı sayılar, 100’den basladıgından 270’inciüç basamaklı sayı, 100 + 270 − 1 = 369’dur. Böylece, 371’inci üç basamaklı sayı370 ve ilk rakamı 3 oldugundan, olusturulan sayının 1000’inci rakamı 3’tür.

c) Önce, 1 2 3 999 sayılarının rakamlarıyla olusturulan sayının rakamları topla­mını bulalım. 0 1 2 9 rakamları ile olusturulacak üç basamaklı bir sayıda herbir basamakta 0 1 2 9 rakamları 100010 kez bulunur. O halde 1 2 3 999sayılarının rakamlarının toplamı

(0 + 1 + · · ·+ 9) ·µ1000

10

¶· 3 = 13500

olur. O halde, geriye 1000 1001 1002 1071 sayılarının rakamları toplamını hesap­lamak kaldı. Binler basamagındaki birlerin toplamı 72’dir. Yüzler basamagındakisayıların toplamı 0’dır. Onlar basamagında ise, 1,2,3,4,5,6 onar kez, 7 ise iki kezkullanıldıgından, bunların toplamı :

(1 + 2 + 3 + 4 + 5 + 6) 10 + 14 = 224

bulunur. Birler basamagında ise, 2 3 9 rakamları 7’ser, 1 rakamı ise 8 defa kul­lanıldıgından, rakamların toplamı

(1 + 2 + 3 + · · ·+ 9) 7 + 1 = 9 · 102

· 7 + 1 = 316olur. Sonuç olarak, 12345678910111210701071 sayısının rakamları toplamı :

13500 + 224 + 316 = 14 040

elde edilir.

Page 18: MATEMATİK OLİMPİYATLARINA HAZIRLIK 1Herbirkitaptaöncelikle,konuya veokonuileilgiliörnek ö˘gretici olabilecek sorulara yerverdim.Dahasonra,herbirkonuileilgilidünyada de˘gi¸

18 Matematik Olimpiyatlarına Hazırlık 1

Alıstırma :

a) 1’den 100’e kadar (1 ve 100 dahil) sayıların yanyana yazılmasıyla elde edilensayının rakamları toplamını bulunuz.b) 1’den 2013’e (1 ve 2013 dahil) kadar sayıların yanyana yazılmasıyla elde edilensayının rakamları toplamını bulunuz.Yanıt :

a) 10 (1 + 2 + 3 + · · ·+ 9) · 2 + 1 = 901b) 13500 + 14500 + 83 = 28 083

Örnek 12 2000’den küçük pozitif tamsayıların herbirinin rakamları toplamı he­saplanıyor ve aynı toplamı veren sayılar arasında daima en küçük olan sayı alı­narak bir küme olusturuluyor.a) Bu kümenin eleman sayısı kaçtır?b) Bu kümedeki tüm sayıların toplamı kaçtır?c) Bu küme 222 elemanlı olsaydı, sayısal degeri en büyük eleman en az kaç olurdu?

Çözüm :

a) Bu kümedeki, rakamları toplamı en büyük olan eleman 1999 olacaktır ki, bu sayınınrakamları toplamı da 28’dir. Buna göre, rakamları toplamı 1 olan, 2 olan, ....,28 olanen küçük elemanları sırasıyla yazarak, kümenin,

= 1 2 8 9 19 29 39 99 199 299 999 1999seklinde olacagı hemen görülebilir. Rakamları toplamı 1,2,3,...,28 degerlerini alabilir.Yani, ’nin eleman sayısı 28’dir.

b) Kümedeki 9’arlı elemanların toplamı,

1 + 2 + · · ·+ 9 = 9 · 102

= 45

19 + 29 + · · ·+ 99 =µ99− 1910

+ 1

¶µ99 + 19

2

¶= 531

199 + 299 + · · ·+ 999 =µ999− 199100

+ 1

¶µ999 + 199

2

¶= 5391

oldugundan, istenen toplam 45 + 531 + 5391 + 1999 = 7966 olur.

c) Küme 222 elemanlı olsaydı, en büyük elemanın rakamları toplamı 222 olurdu.222 = 9 · 24+6 oldugundan, en büyük eleman, 24 tane 9 olmak üzere 69999 sayısıolurdu.

Page 19: MATEMATİK OLİMPİYATLARINA HAZIRLIK 1Herbirkitaptaöncelikle,konuya veokonuileilgiliörnek ö˘gretici olabilecek sorulara yerverdim.Dahasonra,herbirkonuileilgilidünyada de˘gi¸

Temel Bilgiler 19

1.3 Bölünebilme Kuralları

1) 2’ye bölünebilme : Sayı çift ise 2’ye bölünür.2) 3’e bölünebilme : Sayının rakamları toplamı 3’ün katı ise 3’e bölünür.3) 4’e bölünebilme : Sayının son iki rakamı 4’e bölünürse sayı 4’e bölünür.4) 5’e bölünebilme : Sayının son rakamı 0 veya 5 ise sayı 5’e bölünür.5) 6’ya bölünebilme . Sayı hem hem 2 hem de 3’e bölünürse 6’ya bölünebilir.6) 7’ye bölünebilme : 10987654321 10 basamaklı sayısını göz önüne

alalım. Bu sayının birler basamagından itibaren rakamlarını sırasıyla 1 3 ve 2 ileçarparız. Daha sonra her bir üçlüyü toplarız. Birinci üçlü + ile ikinci üçlü − ileüçüncü üçlü+ ile çarpılır ve bulunan sayılar toplanır. Elde edilen sayı 7’ye bölünürsesayı 7’ye bölünür.

(1 + 32 + 23)− (4 + 35 + 26) + (7 + 38 + 29)− (10)Örnegin 4292 736 sayısı için (6 + 3 · 3 + 2 · 7)− (2 + 3 · 9 + 2 · 2) + 4 = 0 oldu­gundan sayımız 7’ye bölünür.

7) 8’e bölünebilme : Sayının son üç rakamı 8’e bölünüyorsa sayı 8’e bölünür.8) 9’a bölünebilme : Sayının rakamları toplamı 9’a bölünüyorsa sayı 9’a bölünür9) 10’a bölünebilme : Sayının son rakamı 0 olmalıdır.10) 11’e bölünebilme : Sayının tek numaralı basamaktaki sayıların toplamı ile

çift numaralı basamaklardaki sayıların toplamının farkı 11 ( ∈ Z) seklinde isesayı 11’e bölünür.

Bir sayının, herhangi iki sayının çarpımına bölünebilmesi için, bu sayıların herbirine bölünebilmesi gerekir. Örnegin, bir sayının 15’e bölünebilmesi için hem 3’ehem de 5’e, 45’e bölünebilmesi için de, hem 5’e ve hem de 9’a tam bölünebilmelidir.

Örnek 13 101, 1001, 10001, 100001, ...,

101 tane

1z | 00...001 sayılarından kaç tanesi

11’e bölünebilir?

Çözüm : 11’e bölünebilme kuralına göre tek sayıda 0 olursa sayı 11’e bölünemeye­cektir. Yani 11’e bölünebilmesi her sayıda çift sayıda 0 olması gerekir. Buna göre2 4 6... 100 sıfır olan sayıların sayısı : (100− 2) 2 + 1 = 50 tane olur.

Örnek 14 A sayısının rakamları toplamı 29, B sayısının rakamları toplamı ise17’dir. Buna göre, A ·B çarpımının 400400 olamayacagını gösteriniz.

Çözüm : sayısının 3’e bölümünden kalan 2 ve sayısının 3’e bölümünden kalan2’dir. O halde, · sayısının 3’e bölümünden kalan 1 olmalıdır. Fakat, 400400sayısının 3’e bölümünde kalan 2 oldugundan, · çarpımı 400400 olamaz.

Page 20: MATEMATİK OLİMPİYATLARINA HAZIRLIK 1Herbirkitaptaöncelikle,konuya veokonuileilgiliörnek ö˘gretici olabilecek sorulara yerverdim.Dahasonra,herbirkonuileilgilidünyada de˘gi¸

20 Matematik Olimpiyatlarına Hazırlık 1

Örnek 15 Bes basamaklı a679b sayısının, 72’ye bölünebilmesi için, a+ b kaçolmalıdır? (KANADA M.O 1980)

Çözüm : 679 sayısı hem 8 hem de 9’a bölünebilmelidir. 8’e bölünme kuralındanson üç basamak 8’in katı olması gerektiginden ve 99 · 8 = 792 oldugundan = 2

olmalıdır. 9’a bölünebilme kuralına göre rakamları toplamı+ 6 + 7 + 9 + 2 = + 24

sayısı 9’un katı olmalı yani = 3 olmalıdır. O halde + = 5 bulunur.

Not : · sayısının bir sayısına bölümünden kalan, ve sayılarının sayısınabölümünden elde edilen kalanların çarpımının ’ye bölünmesiyle elde edilen kalanaesittir.

Örnek 16 101 · 1001 · 10001 · · ·101 tane

1z | 00...001 çarpımının 9’a bölümünden kalan

kaçtır?

Çözüm : Çarpımdaki her bir çarpanın 9’a bölümünen kalan 2’dir. 101 tane çarpanoldugundan, verilen çarpımın 9’a bölümünden kalan

2 · 2 · · · 2 = 2101sayısının 9’a bölümünden kalana esittir. 26 = 64’ün 9’a bölümünden kalan 1 oldugunukullanalım. 101 = 16 · 6 + 5 oldugundan,

2101 = 26 · 26 · · · 26 · 25

sayısının 9’a bölümünden kalan 25’in 9’a bölümünden kalana esit olacaktır. Bu ise,5’tir.

Örnek 17 100’den küçük kaç n dogal sayısı için 4n2+12n− 4 ifadesi 13’ebölünür?

Çözüm : Bir sayı 13’e bölünüyorsa, 13 fazlası da bölünmelidir. Buna göre,42 + 12− 4 + 13 = 42 + 12+ 9 = (2+ 3)2

sayısı da 13’e bölünmelidir. Bunun için 2 + 3 sayısı 13’ün katı olmalıdır. 2 + 3sayısı tek sayı oldugundan, 2+ 3 sayısı,

13 39 65 91 117 143 169 195

olabilir. Bu sayılardan sırasıyla, sayısı5 18 31 44 57 70 83 96

olabilir.

Page 21: MATEMATİK OLİMPİYATLARINA HAZIRLIK 1Herbirkitaptaöncelikle,konuya veokonuileilgiliörnek ö˘gretici olabilecek sorulara yerverdim.Dahasonra,herbirkonuileilgilidünyada de˘gi¸

Temel Bilgiler 21

Örnek 18 Altı basamaklı aabbaa sayısı 7’ye bölünecek sekilde kaç (a, b) ikilisivardır?

Çözüm : 7’ye bölünebilme kuralına göre,(1+ 3+ 2)− (1+ 3+ 2) = −

sayısının 7’nin katı olması gerekir. Buna göre,− = 0 veya − = ±7

olabilir. − = 0 ise = ’dir ve = ∈ 1 2 3 4 5 6 7 8 9 olabilir. Yani,9 tane sayı 7’ye bölünür. − = ±7 ise, bes durum olabilir. = 7 = 0 = 8 = 1; = 1 = 8 ve = 9 = 2; = 2 = 9 durumları. Buradan da, 770077118811 881188 ve 229922 992299 sayıları da 7’ye bölünecektir. Sonuç olarak 14tane ( ) ikilisi vardır.

Örnek 19 1212, 1313, 1414, ..., 4949 sayılarından kaç tanesi 13’e bölündü­günde 10 kalanını verir?

Çözüm : formundaki bir sayıyı : = · 101

seklinde yazabiliriz. Buna göre, sayısının 13’e bölümünden kalanı bulmak için, sayısının 13’e bölümünden kalan sayı ile, 101 sayısının 13’e bölümünden kalansayıyı çarpmalıyız. Buna göre, 101 sayısının 13’e bölümünden kalan 10 oldugundan, sayısının 13’e bölümünden kalan 1 olması gerekir. Bu ise sadece sayısının,

14 27 40

olması durumunda mümkündür. Yanıt 3.

Örnek 20 x,y,z,n ve m rakam olmak üzere, bes basamaklı xyz1n sayısıyla, üçbasamaklı 234 sayısının çarpımı 332m842 olduguna göre, x+y+z+n+m =?

Çözüm : Ikinci çarpanın rakamları toplamı 9’un katı oldugundan 9’a tam bölünürdolayısıyla çarpım da bölünmelidir. Buna göre = 5 olur.

3325842

234= 14213

oldugundan = 1 = 4 = 2 ve = 3 olur. Böylece+ + + + = 15

olur.

Page 22: MATEMATİK OLİMPİYATLARINA HAZIRLIK 1Herbirkitaptaöncelikle,konuya veokonuileilgiliörnek ö˘gretici olabilecek sorulara yerverdim.Dahasonra,herbirkonuileilgilidünyada de˘gi¸

22 Matematik Olimpiyatlarına Hazırlık 1

Örnek 21 10’dan itibaren iki basamaklı bir sayının n kez yanyana yazılmasıylaelde edilen, 2n basamaklı sayıyıa ile gösterelim. Buna göre, a1=10, a2 = 1111,a3 = 121212, 4 = 13131313 , ... seklinde bir sayı dizisi olusacaktır. Bunagöre, a1+a2+a3+ · · ·+ a50 toplamının 7’ye bölümünden kalan kaçtır?

Çözüm : sayısını olusturan iki basamaklı sayı ile ’nin farkı daima 9’dur.1 = 10⇒ 10− 1 = 9; 2 = 1111⇒ 11− 2 = 9;

Buna göre, 50’nci sayı, yani 50 sayısı, 59 sayılarından olusacaktır. Diger yandan, 6basamaklı formundaki bir sayı daima 7’ye bölünür. Gerçekten,

(1+ 3+ 2)− (1+ 3+ 2) = 0oldugu görülebilir. Buna göre, 1+ 2+ 3+ · · ·+50 toplamının 7’ye bölümündenkalan,

10 + 1111 + 13 + 1414 + 16 + 1717 + 19 + 2020 + · · ·+ 58 + 5959toplamının 7’ye bölümünden kalana esit olacaktır. Bu sayılar arasındaki 4 basamaklısayılar formundadır ve

= · 101yazalım. 101’in 7’ye bölümünden kalan 3’tür. O halde, sayısının 7’ye bölümün­den kalan, 3 · sayısının 7’ye bölümünden kalana esit olacaktır. Böylece, istenenkalan

(10 + 13 + 16 + · · ·+ 58) + 3 · (11 + 14 + 17 + 20 + · · ·+ 59)toplamının 7’ye bölümünden kalan olacaktır.µ

58− 103

+ 1

¶µ58 + 10

2

¶+ 3

µ59− 113

+ 1

¶µ59 + 11

2

¶= 2363

sayısının 7’ye bölümünden kalan ise 4’tür.

Örnek 22 Rakamları birbirinden farklım35n dört basamaklı sayısı, 36’ya bölün­dügünde 23 kalanını verdigine göre,m+ n toplamı kaçtır?

Çözüm : Verilenlere göre,35 = 36 + 23 = 4 (9 + 5) + 3 = 9 (4 + 2) + 5

seklinde yazılabilir. Yani, 35 sayısı 4’e bölündügünde 3 kalanını, 9’a bölündügündede 5 kalanını vermelidir. 4’e bölündügünde 3 kalanını verebilmesi = 5 ve = 9

iken mümkündür. Fakat rakamlar birbirinden farklı oldugundan = 9 olmalıdır.Buna göre,

359 iken, 9’a bölündügünde 5 kalanını vermesi için = 6;

olmalıdır. Böylece, + = 6 + 9 = 15 bulunur.

Page 23: MATEMATİK OLİMPİYATLARINA HAZIRLIK 1Herbirkitaptaöncelikle,konuya veokonuileilgiliörnek ö˘gretici olabilecek sorulara yerverdim.Dahasonra,herbirkonuileilgilidünyada de˘gi¸

Temel Bilgiler 23

Örnek 23 S = 7 + 77 + 777 + 7777 + · · ·+100 tanez | 777...77 toplamının 9’a bölü­

münden kalan kaçtır?

Çözüm : Ilk terimin 9’a bölümünden kalan 7’dir. Bundan sonraki her bir terimin9’a bölümünde kalan, kendinden önce gelen terimin 9’a bölümünden kalandan 7 fazlaolacaktır. Buna göre, kalanlar, sırasıyla

7 5 3 1 8 6 4 2 0 7 5

seklinde devam edecektir. Yani, 9 farklı kalan elde edilecektir. 100 = 9 · 11 + 1oldugundan, verilen sayının 9’a bölümünden kalan,

11 (0 + 1 + 2 + 3 + 4 + 5 + 6 + 7 + 8) + 7 = 403

sayısının 9’a bölümünden kalan esittir. O halde, kalan 7 bulunur.

Örnek 24 234’e bölündügünde 11 kalanını veren ve iki asal sayının toplamı veyafarkı seklinde yazılan kaç tane pozitif tamsayı vardır?

Çözüm : = 234 + 11 = + olsun. 234 + 11 tek sayı oldugu için ve asalsayılarından biri tek biri çift olmalıdır. O halde, biri 2 olmalıdır. = 2 alalım. Budurumda = 234 + 9 = 9 (26 + 1) olur ki, bu sayı 9’a bölüneceginden asal sayıolamaz.Simdi ise = 234 + 11 = − olsun. Benzer sekilde = 2 olmak zorundadır.Buna göre,

= 234 + 13 = 234 + 13 = 13 (18 + 1)

olursa = 0 ve = 13 bulunur. Böylece , 11 bulunur. O halde istenen kosullarısaglayan sadece bir pozitif sayı vardır, bu sayı da 11’dir.

Örnek 25 5’in katı olmayan herhangi n tamsayısının karesinin bir fazlasının 5’ebölümünden elde edilebilecek kaç farklı kalan vardır?Çözüm : 5’e bölünmeyen tamsayıları = 5 + 1 = 5 + 2 = 5 + 3 veya = 5+4 seklinde yazabiliriz. olabilir. Hatta, 5+4 yerine 5−1 ve 5+3 yerinede 5− 2 yazılarak, 5’e bölünmeyen tamsayıları kısaca, = 5± 1 ve = 5± 1seklinde gösterebiliriz.Buna göre,

= 5 ± 1 için, 2 + 1 = (5 ± 1)2 + 1 = 252 ± 10 + 2 = 5+ 2, = 5 ± 2 için, 2 + 1 = (5 ± 2)2 + 1 = 252 ± 20 + 5 = 5

oldugundan, sadece 0 ve 2 kalanları elde edilebilir.

Page 24: MATEMATİK OLİMPİYATLARINA HAZIRLIK 1Herbirkitaptaöncelikle,konuya veokonuileilgiliörnek ö˘gretici olabilecek sorulara yerverdim.Dahasonra,herbirkonuileilgilidünyada de˘gi¸

24 Matematik Olimpiyatlarına Hazırlık 1

Örnek 26 n2+n+ 1 sayısının tüm rakamlarının toplamı 101 olacak sekilde kaçtane n dogal sayısı vardır?

Çözüm : Bir dogal sayısı için 2++1 sayısının 3’e bölündügünde hangi kalanlarıelde edebilecegimizi görelim. = 3 formunda olursa, 92 + 3 + 1 olacagından, 3’e bölümünden kalan 1 olur. = 3 + 1 formunda olursa,

(3 + 1)2+ (3 + 1) + 1 = 3

¡32 + 3 + 1

¢oldugundan kalan 0’dır. = 3 − 1 formunda olursa,

(3 − 1)2 + (3 − 1) + 1 = 92 − 3 + 1oldugundan kalan 1’dir.Yani, asla 2 kalanı elde edilemez. 2 + + 1 sayısının 3’e bölümünden 2 kalanıelde edilemiyorsa, rakamları toplamını 3’e böldügümüzde de 2 kalanı elde edemeyiz.Halbuki, 101 sayısının 3’e bölümünden kalan 2’dir. O halde, 2 + + 1 sayısınınrakamları toplamı 101 olacak sekilde dogal sayısı yoktur.

Örnek 27 1’den 100’e kadar (1 ve 100 dahil) tamsayılardan 51 tanesi seçiliyor.Bu seçilen sayılardan bir digerini bölecek sekilde iki sayının mutlaka bulunacagınıgösteriniz.Çözüm : Herhangi bir pozitif tamsayıyı, bir tek sayı olmak üzere, 2 formundayazabiliriz. 2’nin kuvvetlerinde = 1 olacaktır. Seçilen sayılar 1’den 100’e kadarolan sayılardan seçileceginden, sayısı 1 ile 100 arasında olacaktır. Yani, sayısı50 tane tek sayı degeri alabilir. Halbuki, biz 51 sayı seçiyoruz. O halde, seçilensayılardan ikisinin degerleri aynı olacaktır. degerleri aynı ise, 2 formundakibu iki sayının ’si küçük olan, ’si büyük olanı kesinlikle bölecektir.(Bu soruda kullanılan yöntem Güvercin Yuvası ilkesi olarak bilinir. Bu ilke ikinciciltte detaylı incelenmistir.)

Page 25: MATEMATİK OLİMPİYATLARINA HAZIRLIK 1Herbirkitaptaöncelikle,konuya veokonuileilgiliörnek ö˘gretici olabilecek sorulara yerverdim.Dahasonra,herbirkonuileilgilidünyada de˘gi¸

Temel Bilgiler 25

1.4 Bir Sayının Pozitif Bölenlerinin Sayısı, Toplamı ve Çarpımı

F Herhangi bir sayısını, 1 2 sayıları birbirinden farklı asal sayılar ve1 2 sayıları da pozitif tamsayılar olmak üzere,

= 11 · 22 · 33 · · · formunda yazabiliriz. Bu yazılısa, sayısının asal çarpanlarına göre yazılısı denir.F Pozitif Bölen Sayısı

Bir sayısının asal çarpanları ile yazılımı = 11 · 22 · 33 · · ·

olsun. Bu durumda sayısının pozitif bölenlerinin sayısı

() = (1 + 1) (2 + 1) (3 + 1) · · · ( + 1)formülüyle bulunur. Bir sayısının pozitif bölenlerinin tek sayı olması için, sayısınıntüm asal çarpanlarının kuvvetlerinin çift sayı olması gerekir. Kısaca, sayısı tamkareolmalıdır. Örnegin, = 243256 sayısının pozitif bölenlerinin sayısı tektir. sayısınıntüm (hem negatif hem de pozitif) bölenlerinin sayısı ise 2 · ()’dir.F Pozitif Bölenlerin Toplamı

sayısının pozitif bölenlerinin toplamı ise

() =1+11 − 11 − 1

2+12 − 12 − 1

3+11 − 13 − 1 · · ·

+1 − 1 − 1

formülüyle bulunur.F Pozitif Bölenlerin Çarpımı

Bir sayısı, bir sayısının pozitif böleni ise,

sayısı da bir pozitif bölendir.

(Örnegin, 3, 15’in pozitif böleni iken, 153= 5 de pozitif bölenidir.) Bu özellik, bize

tüm pozitif bölenlerin çarpımını kolayca hesaplayabilmemizi saglar. Çünkü, ve

bölenlerin çarpımı, ’dir.* Sayı tamkare degilse, bölenlerin çarpımı bölen sayısının yarısı kadar ’ninkendisiyle çarpılmasıyla elde edilir.Yani, ç() = ()2 olur.

Örnegin, 12 sayısının pozitif bölenlerinin çarpımı 1 · 2 · 3 · 4 · 6 · 12 = 123’dür. Yani,ç(12) = 1262’dir.3* Sayı tamkare ise, bölenlerin çarpımı, bölen sayısının 1 eksiginin yarısı kadar’nin kendisiyle çarpımıyla ’nin karekökünün çarpımına esittir. Yani, ç() =(()−1)2

√ ’dir.

Örnegin, 16 sayısının pozitif bölenlerinin çarpımı da,1 · 2 · 4 · 8 · 16 = 162 · 4 = 1024

dür. Yani, ç(16) = 16(5−1)2√16 = 1024’tür.

Page 26: MATEMATİK OLİMPİYATLARINA HAZIRLIK 1Herbirkitaptaöncelikle,konuya veokonuileilgiliörnek ö˘gretici olabilecek sorulara yerverdim.Dahasonra,herbirkonuileilgilidünyada de˘gi¸

26 Matematik Olimpiyatlarına Hazırlık 1

Örnek 28 48 sayısının pozitif bölenlerinin sayısını, toplamını ve çarpımını bu­lunuz.Çözüm : 48 = 243 oldugundan, pozitif bölen sayısı

(48) = (4 + 1) (1 + 1) = 10

pozitif bölenlerinin toplamı,

(48) =25 − 12− 1 ·

32 − 13− 1 = 124

pozitif bölenlerinin çarpımı ise, bölen sayısı 10 oldugundan,ç(48) = 48102 = 485

olur.

Örnek 29 3600 sayısının pozitif bölenlerinden çift olanlarının sayısını bulunuz.Pozitif çift tamsayı bölenlerinin toplamını bulunuz.Çözüm : 3600 = 245232 oldugundan, toplam 5 · 3 · 3 = 45 pozitif böleni vardır. Tekolanlar ise, 53 formundadır. Bunların sayısı da, 3 · 3 = 9 oldugundan, çift olanlarınsayısı 45− 9 = 36 olur.Pozitif çift tamsayı bölenlerinin toplamı ise,¡

2 + 22 + 23 + 24¢ ¡1 + 5 + 52

¢ ¡1 + 3 + 32

¢= 30 · 31 · 13

çarpımına esittir. Buradan, 12 090 bulunur.

Örnek 30 Pozitif bölenlerinin sayısı tek sayı olan,2013’ten büyük en küçük tam­sayı kaçtır?Çözüm : Sayının bölen sayısının tek sayı olması için, sayı tamkare olmalıdır. Bunagöre, 452 = 2025 oldugundan istenen sekildeki en küçük tamsayı 2025’tir.

Örnek 31 1013’i böldügünde 13 kalanı elde edilen kaç tane pozitif çift tamsayıvardır?Çözüm : 1013 = · + 13 ve 13 yazılabilir. Buradan · = 1000 = 2353

esitligine göre(3 + 1) (3 + 1) = 16

tane pozitif bölen vardır. Bu bölenler arasından 13 kalanından büyük olmayanlarıçıkarmalıyız. Çünkü, bu sayılar, bir sayıyı böldügünde 13 kalanını veremez. 1 2 510 çıkarılırsa 16 − 4 = 12 tane istenen sekilde pozitif tamsayısı vardır. Fakat,soruda pozitif çift sayı denildiginden, 125 çarpanını çıkarmalıyız. Buna göre, 1013’üböldügünde 13 kalanı elde edilen 12− 1 = 11 çift sayı vardır.

Page 27: MATEMATİK OLİMPİYATLARINA HAZIRLIK 1Herbirkitaptaöncelikle,konuya veokonuileilgiliörnek ö˘gretici olabilecek sorulara yerverdim.Dahasonra,herbirkonuileilgilidünyada de˘gi¸

Temel Bilgiler 27

Örnek 32 48 tane pozitif böleni olan en küçük pozitif tamsayı kaçtır?Çözüm : En küçük sayıyı bulabilmek için, asal çarpanları mümkün oldugu kadarküçük asallardan seçeriz. Diger taraftan seçtigimiz bu asal sayıların üslerinin 1 faz­lalarının çarpımları 48 olmalıdır.

= 2357

oldugunu farzedelim. 48 = 16·3 oldugundan, üslerden biri mutlaka 2 olmalıdır. Bunagöre, = 3 = 2 ve = = 1 alınırsa, = 23 ·32 ·5·7 = 2520 bulunur. Bu sayınınpozitif bölenlerinin sayısı gerçekten, (3 + 1) (2 + 1) (1 + 1) (1 + 1) = 48’dir.

Örnek 33 2 ve 3’e bölünen pozitif bir tamsayının tam 33 pozitif böleni varsa, busayı en küçük kaç olabilir?Çözüm : Pozitif bölenlerinin sayısı 33 ve 33 = 3 · 11 oldugundan istenen sayının 2ve 3’ten baska asal böleni olamaz. Buna göre, istenen sekildeki en küçük sayı 210 · 32elde edilir.

Örnek 34 108000 sayısının pozitif bölenlerinin kaç tanesi 8’e bölünür ama 9’abölünmez?Çözüm : 108000 = 253353 biçiminde asal çarpanlarına ayrılabilir. Buna göre, iste­digimiz pozitif bölenler, 235 formundaki sayılar arasından 3 ≤ ≤ 5 0 ≤ ≤ 1ve 0 ≤ ≤ 3 kosulunu saglayan sayılardır. Dolayısıyla, istenen sekildeki sayılarınsayısı : 3 · 2 · 4 = 24 olarak bulunur.

Örnek 35 12 +22+32+ · · ·+ n2 =n (n+ 1) (2n+ 1)

6olduguna göre,

12−1 + 22−2 + 32 −3 + · · ·+ 1002−100ifadesinin kaç tane tek sayı pozitif böleni vardır?

Çözüm : Verilen sayıyı,¡12 + 22 + · · ·+ 1002¢− (1 + 2 + · · ·+ 100) =

100 · 101 · 2016

− 100 · 1012

= 50 · 101 (67− 1)= 22 · 3 · 52 · 11 · 101

biçiminde hesaplayabiliriz. Buna göre, tek sayı pozitif bölenlerinin sayısı,(1 + 1) (2 + 1) (1 + 1) (1 + 1) = 24

olarak bulunur.

Page 28: MATEMATİK OLİMPİYATLARINA HAZIRLIK 1Herbirkitaptaöncelikle,konuya veokonuileilgiliörnek ö˘gretici olabilecek sorulara yerverdim.Dahasonra,herbirkonuileilgilidünyada de˘gi¸

28 Matematik Olimpiyatlarına Hazırlık 1

Örnek 36 992+1, 992+2, ..., 1042−2, 1042−1, 1042 sayılarından 99’a bölü­nenlerin toplamının, kaç pozitif böleni vardır?

Çözüm : Çözümümüzü en genel halde yapalım. Herhangi,2 + 1 2 + 2 (+ 5)

2

sayı dizisinde, Terim Sayısı = Son Terim − Ilk Terim +1 oldugu kullanılarak,(+ 5)

2 − ¡2 + 1¢+ 1 = 10+ 25sayı oldugu görülebilir. Buna göre, 25 için, verilen sayıların arasından ilebölünenler sadece

2 + 2 + 2 2 + 3 2 + 10

sayılarıdır. Bunların toplamı da,¡2 +

¢+¡2 + 2

¢+ · · ·+ ¡2 + 10¢ = 55+ 102 = 5 (2+ 11)

elde edilir. Böylece, = 99 için, toplam5 (2+ 11) = 5 · 99 · (2 · 99 + 11) = 32 · 5 · 112 · 19

elde edilir. Buna göre, pozitif bölenlerinin sayısı :(2 + 1) (1 + 1) (2 + 1) (1 + 1) = 36

bulunur.

Page 29: MATEMATİK OLİMPİYATLARINA HAZIRLIK 1Herbirkitaptaöncelikle,konuya veokonuileilgiliörnek ö˘gretici olabilecek sorulara yerverdim.Dahasonra,herbirkonuileilgilidünyada de˘gi¸

Temel Bilgiler 29

1.5 EBOB - EKOK

I Iki veya daha fazla sayının her birini bölebilen en büyük pozitif tamsayıya busayıların ortak bölenlerinin en büyügü denir. iki tamsayı olmak üzere ve

sayılarının ortak bölenlerinin en büyügü EBOB ( ) veya bazen kısaca ( ) ilegösterilir. EBOB ( ) = 1 ise ve sayılarına aralarında asal sayılar denir.Ardısık iki pozitif sayı aralarında asaldır.

Örnek 37 Kareleri 2 ile 200 sayıları arasında olan, herhangi ikisi aralarında asalolacak sekilde en fazla kaç sayı seçilebilir?

Çözüm : 142 = 196 oldugundan, kareleri 2 ile 200 arasında olan sayılar 2, 3, 4, 5, 6,7, 8, 9, 10, 11, 12, 13, 14’tür Bu sayıların arasından herhangi ikisinin aralarında asalolması için,

2 3 5 7 11 13

asal sayılarını seçebiliriz. Eger 7 sayı seçilirse iki sayının asal çarpanları arasında buasal sayılardan biri ortak olacaktır ki bu da aralarında asal olma kosulunu bozacaktır.O halde en fazla 6 sayı seçilebilir.

Iki sayının EBOB’unu bulmak için, sayılar asal çarpanlarına ayrılır ve her ikisayıda da bulunan ortak asal çarpanlardan, üssü küçük olanlar çarpılır.

Örnegin,EBOB

¡243252 712233

¢= 2232 = 36

seklinde bulunur. Yani, ortak asal çarpanlardan, üssü en küçük olanları çarptık.

Örnek 38 4050’nin böleni olup 5040’ın böleni olmayan kaç pozitif tamsayı vardır?

Çözüm : 4050 = 2150550 oldugundan 4050 sayısının, (150 + 1) (50 + 1) tane pozitifböleni vardır. Bu bölenlerden, 5040’ın bölenleri olanları çıkaralım. Bunun için, herikisinin bölenlerinin sayısını bulmalıyız. 5040 = 240580 ve

EBOB¡5040 4050

¢= 240550

oldugundan, her ikisinin de böleni olan (40 + 1) (50 + 1) pozitif tamsayı vardır. Ohalde, 4050’nin böleni olup 5040’ın böleni olmayan,

151 · 51− 41 · 51 = 51 · 110 = 5610pozitif tamsayı vardır.

Page 30: MATEMATİK OLİMPİYATLARINA HAZIRLIK 1Herbirkitaptaöncelikle,konuya veokonuileilgiliörnek ö˘gretici olabilecek sorulara yerverdim.Dahasonra,herbirkonuileilgilidünyada de˘gi¸

30 Matematik Olimpiyatlarına Hazırlık 1

Örnek 39 504 veya 203 sayılarını bölen kaç pozitif tamsayı vardır?Çözüm : Sorudaki veya baglacı çok önemli bir ayrıntıdır. 504 = 2458 sayısınınpozitif bölenlerinin sayısı 5 · 9 = 45 203 = 2653 sayısının pozitif bölenlerinin sayısıise, 7 · 4 = 28’dir. Fakat, cevap 45 + 28 = 73 degildir. Çünkü, 73 sayısının içinde,504 ve 203 sayılarının her ikisini de bölen sayılar, iki kez hesaplanmıstır. O halde, 504ve 203 sayılarının her ikisini de bölen sayıların sayısını çıkarmalıyız. Her ikisini debölen sayılar, aslında EBOB’unu bölen sayılardır.

EBOB¡504 203

¢=EBOB

¡2458 2653

¢= 2453

oldugundan, her ikisini de bölenlerin sayısı (4 + 1) (3 + 1) = 20 olur. O halde,istenen yanıt : 73− 20 = 53 bulunur.

Örnek 40 EBOB’ları 48, toplamları ise 576 olacak sekilde kaç pozitif tamsayıikilisi vardır?Çözüm : Bu iki sayıya ve diyelim. ve aralarında asal olmak üzere, = 48ve = 48 biçiminde yazılabilir. Toplamları 576 ise, + = 48 (+ ) = 576

esitliginden, + = 12 olacaktır. ve aralarında asal olacagından (1 11) veya(5 7) ikililerinden biri olabilir. Bu durumda, bu sayılar 48 528 veya 240 336 ikilileriolabilir.

F Iki sayının ortak böleni, bu sayıların tamsayı katlarının farkını da böler. Buözelligi kullanarak iki sayının EBOB’unu bulmak mümkündür. (Öklid Algoritması)

Örnek 41 EBOB(10055, 22121) =?Çözüm : Küçük olan sayının bir tamsayı katını büyük olan sayıdan çıkaracagız. Bu­rada, 2 katını çıkarmamız yeterli. Buna göre, 22121− 2 · 10055 = 2011 oldugundan,

EBOB(10055 22121) =EBOB(10055 2011)

bulunur. 10055 sayısı 2011 sayısının 5 katı oldugundan, her iki sayı da 2011’e bölü­necektir. Yani, EBOB(10055 22121) = 2011 bulunur.

Örnek 42 n bir pozitif tamsayı olmak üzere, 3n+5 ve 7n+8 sayılarının EBOB’ukaç farklı pozitif tamsayı olabilir?Çözüm : Bu iki sayının EBOB’u 3+5 ile (7+ 8)−2 (3+ 5) = −2 sayılarınınEBOB’una esittir. O halde, 3+5 ile −2 sayılarının EBOB’unu bulmamız yeterlidir.Bu iki sayının EBOB’u ise − 2 sayısı ile 3 + 5 − 3 (− 2) = 11 sayılarınınEBOB’una esittir. Dolayısıyla 3 + 5 ve 7 + 8 sayılarının EBOB’u ya 1 veya 11olabilir. EBOB’un 11 olması için, − 2 sayısının 11’e bölünmesi gerekir. Örnegin, = 13 ise sayılar, 3 ·13+5 = 44 ve 7 ·13+8 = 99 olur ki, bunların EBOB’u 11’dir.

Page 31: MATEMATİK OLİMPİYATLARINA HAZIRLIK 1Herbirkitaptaöncelikle,konuya veokonuileilgiliörnek ö˘gretici olabilecek sorulara yerverdim.Dahasonra,herbirkonuileilgilidünyada de˘gi¸

Temel Bilgiler 31

Örnek 43 n sayısı 2011’den büyük bir tamsayı olduguna göre, 11n+9 ve 3n+2aralarında asal olmayacak sekilde en küçük sayısını bulunuz.Çözüm : Bu iki sayının EBOB’u 3+2 ile 11+9− 3 (3+ 2) = 2+3 sayısınınEBOB’una esittir. Bunların EBOB’u da 2 + 3 ile 3 + 2 − (2+ 3) = − 1sayılarının EBOB’una esittir. O halde, EBOB(− 1 2+ 3)’ü bulmalıyız.

EBOB(− 1 2+ 3) =EBOB(− 1 2+ 3− 2 (− 1)) =EBOB(− 1 5)oldugundan 5 olabilir. Bunun için, − 1 sayısı 5’in katı olmalıdır. sayısı 2011’denbüyük oldugundan, en küçük sayısı için, − 1 = 2015 ve = 2016 alınabilir.

Örnek 442n− 35n− 1 kesiri, 1’den büyük bir n tamsayısı için, a pozitif sayısı ile

sadelestirilebildigine göre, a sayısı kaçtır?

Çözüm :2− 35− 1 kesiri sayısı ile sadelestirilebildgine göre, sayısı, 2 − 3 ve

5− 1 sayılarının her ikisinin de bölenidir. Buna göre, sayısı,(5− 1)− 2 (2− 3) = + 5

sayısını da bölmelidir. Benzer düsünce ile, sayısı, hem 2−3 hem de, +5 sayısınıbölüyorsa,

2 (+ 5)− (2− 3) = 13sayısını da bölmelidir. O halde, sayısı 13’tür.

I Iki veya daha fazla sayının her birine bölünen en küçük pozitif tamsayıya busayıların ortak katlarının en küçügü denir. iki tamsayı olmak üzere ve

sayılarının ortak katlarının en küçügü EKOK( ) veya kısaca [ ] ile gösterilir.Iki sayının EKOK’unu bulmak için, sayılar asal çarpanlarına ayrılır ve sayılar-daki asal çarpanlardan üssü en büyük olanlar çarpılarak EKOK bulunur.

Örnegin,EKOK

¡243252712233

¢= 2433527 = 75 600

seklinde bulunur. Yani, her asal çarpandan, üssü en büyük olanları çarptık.

Örnek 45 391 sayısı 12 tane pozitif tamsayının toplamı olduguna göre bu 12sayının EKOK’u en küçük kaç olabilir?Çözüm : 12 · 33 = 396 oldugundanEKOK’un 33 ve 33’ten küçük olması mümkündegildir. Buna göre EKOK acaba 34 olabilir mi? EKOK = 34 olması için sayılar34 17, 2 ve 1’den olusmalıdır. 11 tane 34 ve 1 tane 17 alınırsa, 34 · 11 + 17 = 391olacagından, EKOK en küçük 34 olabilir.

Page 32: MATEMATİK OLİMPİYATLARINA HAZIRLIK 1Herbirkitaptaöncelikle,konuya veokonuileilgiliörnek ö˘gretici olabilecek sorulara yerverdim.Dahasonra,herbirkonuileilgilidünyada de˘gi¸

32 Matematik Olimpiyatlarına Hazırlık 1

I Iki sayının EKOK’ı ile EBOB’larının çarpımı, bu iki sayının çarpımına esittir.Yani,

· = EKOK ( ) ·EBOB [ ]esitligi saglanır.

Örnek 46 En küçük ortak katları 144 olan kaç farklı iki pozitif tamsayı buluna­bilir? (Örnegin, bir tanesi (16, 18) ikilisidir. EKOK’ları 144’tür.)Çözüm : Bu iki sayıyı ve ile gösterelim. 144 = 2432 oldugundan,

= 2 · 3 ve = 2 · 3

formunda olacaktır. EKOK’un tanımı geregi, ve ’in en büyügü 4, ve ’nin en büyügü 2 olmalıdır.

O halde, bunları kaç farklı sekilde seçebilecegimizi görelim.( ) ikilisi, (0 4) ; (1 4) ; (2 4) ; (3 4) ; (4 4) ; (4 3) ; (4 2) ; (4 1) ;

(4 0) olacak sekilde 9 farklı sekilde olabilir.( ) ikilisi, (0 2) ; (1 2) ; (2 2); (2 1) ; (2 0) olacak sekilde 5 farklı sekilde

olabilir.Sonuç olarak, bu iki gruptaki, her gruptan seçilen, her bir ikili için farklı iki sayı eldeedileceginden, 9 · 5 = 45 farklı iki pozitif tamsayı bulunabilir.Örnegin, ( ) = (4 2) ( ) = (1 2) alınırsa,

= 24 · 31 = 48 ve = 22 · 32 = 36elde edilir. EKOK(48 36) = 144’dür.

Alıstırma : Siz de, EKOK() = 233252 olacak sekilde 175 tane () pozitiftamsayı ikilisi oldugunu görünüz.

Örnek 47 a ve b dogal sayılarının ortak bölenlerinin en büyügü 9, ortak kat­larının en küçügü ise 270’dir. Buna göre, bu iki sayının toplamı en küçük kaçolabilir? (Dogus Ün. Fen Liseleri Y. 2005)Çözüm : EBOB( ) ·EKOK ( ) = · oldugundan,

· = 9 · 270 = 35 · 2 · 5olur. Bu iki sayının olabilecek en küçük toplamını bulmak için, ve ’yi birbirineen yakın olacak sekilde seçmeliyiz. Her iki sayının da 9’a bölünebildigi de gözönünealınırsa, = 9 · 5 ve = 9 · 6 seçilebilir. Bu durumda en küçük toplam 45+ 54 = 99

olarak bulunur.

Page 33: MATEMATİK OLİMPİYATLARINA HAZIRLIK 1Herbirkitaptaöncelikle,konuya veokonuileilgiliörnek ö˘gretici olabilecek sorulara yerverdim.Dahasonra,herbirkonuileilgilidünyada de˘gi¸

Temel Bilgiler 33

Örnek 48 En küçük ortak katları 120 ve en büyük ortak bölenleri 10 olan kaçfarklı sayı ikilisi vardır?

Çözüm : EBOB( ) ·EKOK ( ) = · oldugundan, · = 1200 = 2 · 2 · 3 · (10) · (10)

olmalıdır. ve ’nin her ikisinde de 10 çarpanı bulunması gerekir. Buna göre, ( )ikilisi, (20 60) ; (40 30) (10 120) durumlarından biri olabilir. Yani, üç farklı sayıikilisi vardır.

Örnek 49 Üç ardısık sayıdan birincisi 3’e, ikincisi 8’e, üçüncüsü 13’e tam bölünü­yor. Bu üç ardısık sayının toplamı en az kaçtır?

Çözüm : 3,8 ve 13 sayıları beser beser artmıstır. Buna göre, uygun bir sayısıylatoplayıp 5’e bölerek bu sayıları ardısık hale getirebiliriz.

3 +

58 +

513 +

5

sayılarının, sırasıyla 3,8 ve 13’e bölünen birer tamsayı olması için, sayısının 3, 8 ve13’ye bölünen bir sayı olması gerekir. Buna göre, en az,

EKOK (3 8 13) = 3 · 8 · 13 = 312alınabilir. Böylece,

3 + 312

5= 63

8 + 312

5= 64 ve

13 + 312

5= 65

sayıları, istenen kosulu saglayan en küçük sayılar olacaktır. O halde bu üç ardısıksayının toplamı en az 63 + 64 + 65 = 192 olarak bulunur.

Örnek 50 Birincisi 3’e, ikincisi 5’e, üçüncüsü 7’ye ve dördüncüsü 9’a tam bölü­nen en küçük dört ardısık sayıyı bulunuz.

Çözüm : Bir önceki örnekte oldugu gibi+ 3

2+ 5

2+ 7

2+ 9

2

sayılarının istenen sekilde olması için = EKOK (3 5 7 9) = 5 · 7 · 9 = 315

alınmalıdır. Böylece sayılar315 + 3

2315 + 5

2315 + 7

2315 + 9

2

yani, 159 160 161 162 olarak bulunur.

Page 34: MATEMATİK OLİMPİYATLARINA HAZIRLIK 1Herbirkitaptaöncelikle,konuya veokonuileilgiliörnek ö˘gretici olabilecek sorulara yerverdim.Dahasonra,herbirkonuileilgilidünyada de˘gi¸

34 Matematik Olimpiyatlarına Hazırlık 1

1.6 Modüler Aritmetik

pozitif bir tamsayı olmak üzere − sayısı ’ye tam bölünebiliyor ise ve

sayıları modül ’ye göre denktirler denir ve ≡ (mod)

seklinde yazılır. Yani ≡ (mod) ifadesinde sayısının ile bölümündenelde edilen kalanı ifade etmektedir. O halde ≡ (mod) ise, ∈ Z olmak üzere, = + olur. ≡ (mod) ifadesine denklik ya da kongruans adı verilir.Örnegin, 100! ≡? (mod 11) matematiksel ifadesini, "100! sayısının 11’e bölümün­den kalan kaçtır?" seklinde ifade edebiliriz. Benzer sekilde, 222 · · · 2| z

100 tane

sayısının 9’a

bölümünden kalanın bulunmasını, 222 · · · 2| z 100 tane

≡? (mod 9) ile ifade edebiliriz. Buna

göre, 9’a bölünme kuralı göz önüne alınırsa,222 · · · 2| z 100 tane

≡ 2 · 100 = 200 ≡ 2 (mod 9)

yazılabilir.Not : Bir sayının son rakamını bulmak için mod10’a, son iki rakamını bulmak içinmod100’e, son üç rakamını bulmak içinmod1000’e vb. bakılır.

Örnek 51 41000 sayısının son rakamı kaçtır?Çözüm : 41000 ≡? (mod 10) denkligini hesaplamalıyız. Bunun için,

41 ≡ 4 (mod 10) 42 ≡ 6 (mod 10) 43 ≡ 4 (mod 10) 44 ≡ 6 (mod 10)olacagından, 4’ün çift kuvvetlerinde son rakam 6, tek kuvvetlerinde ise son rakam4’tür. O halde, 41000 sayısının son rakamı 6 olur.

Örnek 52 20112012 sayısının 7’ye bölümünden kalan nedir?Çözüm : 2011 = 7 · 287 + 2 oldugundan, dolayı

20112012 ≡ 22012 (mod 7)yazabiliriz. Simdi, 2’nin kuvvetlerini mod7’de inceleyerek, 1 veya periyodik olun­caya devam edelim.

21 ≡ 2 (mod 7) 22 ≡ 4 (mod 7) 23 ≡ 1 (mod 7)oldugundan, 2’nin 3’ün katı olan tüm kuvvetlerimod7’de 1 kalanını verecektir. Bunagöre,

22012 = 22010 · 22 = ¡23¢670·3 · 22 ≡ 1 · 4 ≡ 4 (mod 7)elde edilir. Yani, kalan 4 olarak bulunur.

Page 35: MATEMATİK OLİMPİYATLARINA HAZIRLIK 1Herbirkitaptaöncelikle,konuya veokonuileilgiliörnek ö˘gretici olabilecek sorulara yerverdim.Dahasonra,herbirkonuileilgilidünyada de˘gi¸

Temel Bilgiler 35

Örnek 53 13 +23 +33 +43 + · · ·+ 20133 sayısının 8’e bölümünden kalankaçtır?

Çözüm : tek ise, 3 ≡ (mod 8) ve çift ise, 3 ≡ 0 (mod 8) oldugundan,istenen kalan

13 + 33 + 53 + · · ·+ 20133

sayısının 8’e bölümünden kalana esittir. Kalanlar 1 3 5 7 seklinde periyodik olarakdevam edeceginden, ve her periyottaki sayıların toplamı 8’e tam bölüneceginden,sadece en son sayılardan dörtlü olusturmayan sayıları incelemek yeterlidir.20133 ≡ 5 (mod 8) oldugundan,

13 + 23 + 33 + 43 + · · ·+ 20133 ≡ 20093 + 20113 + 20133

≡ 1 + 3 + 5

≡ 1 (mod 8)

elde edilir.

Örnek 54 2+27 sayısı 7’ye bölünecek sekilde 100’den küçük kaç pozitifn tam­sayısı vardır?

Çözüm : 21 ≡ 2 (mod 7) 22 ≡ 4 (mod 7) 23 ≡ 1 (mod 7)

oldugundan = 3 oldugunda,2 + 27 ≡ 1 + 27 ≡ 0 (mod 7)

olur. O halde 100’den küçük 3 formunda olan sayıların sayısını bulmalıyız. Bunlar,3 6 9 99

sayıları oldugundan, (99− 3) 3 + 1 = 33 tanedir.

Örnek 55 Ilk 98 sayının 1234...9798 biçiminde art arda yazılmasıyla elde edilensayının 45’e bölümünden kalan kaçtır?

Çözüm : Verilen sayının rakamları toplamı

1 + 2 + 3 + · · ·+ 98 = 98 · 992

= 99 · 49oldugundan, sayı 9’a tam bölünür. Sayının son rakamı 8 oldugundan, 5’e bölündügünde,3 kalanını verir. Buna göre, kalanın 9’un katı ve 5’e bölündügünde 3 kalanını verenbir sayı olması gerekir. Bu sayı 18’dir.

Page 36: MATEMATİK OLİMPİYATLARINA HAZIRLIK 1Herbirkitaptaöncelikle,konuya veokonuileilgiliörnek ö˘gretici olabilecek sorulara yerverdim.Dahasonra,herbirkonuileilgilidünyada de˘gi¸

36 Matematik Olimpiyatlarına Hazırlık 1

Örnek 56 1 ile 701 arasında alınan pozitif tamsayılarla olusturulan (x, y) sıralıikililerinden kaçı için x2+y2 sayısı 49’a kalansız bölünür? (Dogus Ün. Mat.Y. ­2007)

Çözüm : 2 + 2 ifadesi 49’a tam bölünüyorsa 7’ye de tam bölünür. Bir sayınınkaresinimod7’de inceleyelim.

02 ≡ 0 (mod 7) 12 ≡ 1 (mod 7) 22 ≡ 4 (mod 7) 32 ≡ 2 (mod 7) 42 ≡ 2 (mod 7) 52 ≡ 4 (mod 7) 62 ≡ 1 (mod 7)

elde edilir. Buna göre, 2 + 2 ifadesinin mod7’de 0 olabilmesi için, yani 7’ye tambölünebilmesi için, hem 2 hem de 2 ifadelerinin 7’ye tam bölünebilmesi gerekir.Bunun için de hem hem de sayıları 7’nin katı olmalıdır. Bu durumda, 2 + 2

sayısının 49’a bölünecegi açıktır. Böylece, 1 ile 701 arasında 7 ile bölünen 100 tane sayısı ve 100 tane de sayısı bulunacagından, istenen sekilde 100 · 100 = 104 tane( ) ikilisi oldugu görülür.

Örnek 57√4k+ 3 sayısı tamsayı olacak sekilde kaç k pozitif tamsayısı vardır?

Çözüm : Verilen ifadenin tamsayı olması için, 4 + 3 sayısının bir tamkare olmasıgerekir. Bu tamkarenin de 4’e bölündügünde 3 kalanını vermesi gerekir. O halde, susoruya cevap arayalım. "Bir sayının karesimod4’de kaç olabilir?".

02 ≡ 0 (mod 4),12 ≡ 1 (mod 4),22 ≡ 0 (mod 4) ,32 ≡ 1 (mod 4)

oldugundan, bir sayının karesi 4’e bölündügünde kalan 0 veya 1 olabilir. Yani, 3olamaz. O halde,

√4 + 3 ifadesi asla bir tamkare olamaz.

Örnek 58 7 ≡ 1 (mod101) denkligini saglayan bir x sayısı bulunuz.

Çözüm : Bu soruyu biraz önceki örneklerde oldugu gibi tek tek hesaplayarak yapmakzordur. Bunu hesaplamıs olsaydık,

71 ≡ 7 (mod 101) 72 ≡ 49 (mod 101) 73 ≡ 40 (mod 101) seklinde devam ederek, 7100 ≡ 1 (mod 101) oldugunu görebilirdik.Açıktır ki, bu bizim oldukça çok vaktimizi alır. Bunun yerine, Fermat Teoremi dedenilen, bir asal sayısı için,

−1 ≡ 1 (mod )denkliginden de yararlanmak daha kolay olabilir. Bu durumda, 7100 ≡ 1 (mod 101)oldugunu dogrudan söyleyebilirdik.

Page 37: MATEMATİK OLİMPİYATLARINA HAZIRLIK 1Herbirkitaptaöncelikle,konuya veokonuileilgiliörnek ö˘gretici olabilecek sorulara yerverdim.Dahasonra,herbirkonuileilgilidünyada de˘gi¸

Temel Bilgiler 37

I Fermat Teoremi : bir asal sayı ise, ≡ (mod )

’dir. Eger, EBOB( ) = 1 ise, −1 ≡ 1 (mod ) olur.

Örnek 59 13200 sayısının 19’a bölümünden kalan kaçtır?Çözüm : Fermat teoreminden, 1318 ≡ 1 (mod 19) diyebiliriz. Buna göre,

13200 =¡1318

¢11 · 132 ≡ 132 ≡ (−6)2 = 36 ≡ 17 (mod 19)elde edilir.

Örnek 60 10100 sayısının 65’e bölümünden kalan kaçtır?Çözüm : 65 = 13·5 oldugundan, 10100 sayısının önce 13’e, sonra da 5’e bölümündenkalanı bulalım. 10100 ≡? (mod 13) denkligini Fermat Teoremini kullanarak bulabili­riz. 1012 ≡ 1 (mod 13) oldugundan,

10100 = 1012·8+4 =¡1012

¢8104 ≡ 104 = 1002 ≡ 42 ≡ 3 (mod 13)

elde edilir. Diger yandan, 10100 sayısının 5’e bölümünden kalan 0’dır. O halde, 10100sayısının 65’e bölümünden kalan sayı, 13’e bölündügünde 3 kalanını veren ve 5’inkatı olan bir sayıdır. 13 + 3 formundaki sayılardan, 5’in katı olan 65’ten küçük teksayı 55’tir. O halde, 10100 sayısının 65’e bölümünden kalan 55 bulunur.F Asagıdakileri Fermat Teoremini Uygulayarak Hesaplayınız.1. 516 ≡? (mod 17) (Yanıt : 1)2. 320 ≡? (mod 17) (Yanıt : 13)3. 781 ≡? (mod 17) (Yanıt : 7)4. 10100 sayısının 33’e bölümünden kalan kaçtır? (Yanıt : 1)5. 11111 sayısının 65’e bölümünden kalan kaçtır? (Yanıt : 31)I Fermat Teoremi bize her zaman denkligi saglayan en küçük üssü vermeyebilir.Daha küçük bir üs varsa Fermat Teoremi ile elde edilen üssü bölmesi gerekir.

Örnek 61 5 ≡ 1 (mod13) denkligini saglayan en küçük n sayısı kaçtır?Çözüm : Fermat teoremine göre, 512 ≡ 1 (mod 13) oldugunu hemen söyleyebiliriz.5’in daha küçük bir üssünde bu denklik saglanıyorsa, bu üssün 12’yi bölmesi gerekir.Gerçekten, 54 ≡ 1 (mod 13)’tür.I Burada aklımıza su soru gelecektir. Fermat teoremini sadece mod, yani sayısıasal iken kullanabiliriz. Eger, mod asal olmazsa ne yapacagız? Bunun da çözümüvar. Bu durumda da Euler Teoremi dedigimiz teoremi kullanacagız. Bu teorem­lerle ilgili detaylı bilgileri ve degisik soruları Matematik Olimpiyatlarına Hazırlık 3kitabında bulabilirsiniz. Burada sadece bu teoremlere kısaca deginilmistir.

Page 38: MATEMATİK OLİMPİYATLARINA HAZIRLIK 1Herbirkitaptaöncelikle,konuya veokonuileilgiliörnek ö˘gretici olabilecek sorulara yerverdim.Dahasonra,herbirkonuileilgilidünyada de˘gi¸

38 Matematik Olimpiyatlarına Hazırlık 1

1.7 Basit Denklem Çözümleri ve Sayıların ÖzelliklerininKullanılması

Sayının tamsayı veya pozitif tamsayı olmasının, tek veya çift olmasının, asalolmasının, ardısık sayılar olmasının asagıdaki denklemlerde nasıl kullanıldıgını ince­leyiniz. Bu örneklerdeki yorumların, sadece verilen sayı kümeleri ve bu kümelerinaltkümeleri için geçerli oldugunu ve baska bir sayı kümesinde benzer yorumlarınyapılamayacagını görünüz.

Örnek 62 n (n+ 1) = p denklemini saglayan tüm p asal sayılarını ve n ve kpozitif tamsayılarını bulunuz.Çözüm : ile + 1 sayıları ardısıktır ve biri tek biri çifttir. O halde, bu denkleminsaglanması için, esitligin sag tarafındaki sayının çift olması gerekir ki, asal oldugun­dan sadece 2 olabilir. Bu durumda, (+ 1) = 2 olur. Bu esitligin saglanabilmesisadece = 1 durumunda mümkündür. Çünkü, 1 durumunda, sol tarafın bir tekçarpanı vardır, oysa sag taraf 2’nin bir kuvvetidir. Buna göre, = 1 yazılırsa, 2 = 2olur. Buradan = 1 elde edilir.

Örnek 63 x, y ve z asal sayılar olmak üzere,½x+ y + z = 20

y + 2x = z

denklem sisteminin kaç çözümü vardır?Çözüm : Üç asal sayının toplamının bir çift sayı olması için sayılardan biri mutlaka2 olmalıdır. Aksi durumda, üçü de tek asal sayı olursa toplam da tek sayı olacaktır.Diger taraftan, bir çift sayı olamaz, çünkü, çift olursa sistemin ikinci denkleminegöre de çift olmak zorunda olur. O halde = 2 olmalıdır. Bu deger denklemlerdeyerine yazılırsa, + = 18 ve + 4 = denklemleri elde edilir. Bu denklemlerinçözümünden, = 11 ve = 7 bulunur.

Örnek 64 n,m ∈ Z+ olmak üzere 5n+ 3m = 100 denklemini çözünüz.Çözüm : Denklemde, 5 ve 100 terimleri 5’in katıdır. O halde, 3 terimi de 5’inkatı olmalıdır. Bu durumda, , 5’in katı olmalıdır. Denklemi daha basit hale getirmekiçin, ∈ Z+ için, = 5 yazalım. Böylece, denklem daha basit olarak,

+ 3 = 20 veya = 20−

3

sekline dönüsür. ve sayıları pozitif tamsayı oldugundan, = 2 5 8 11 14 veya17 olabilir.Not : Burada, pozitif tamsayılar kümesi yerine tamsayılar kümesi alınsaydı, sonsuzsayıda sayısı elde ederdik. Yani, denklemin verildigi sayı kümesi problemdeki enönemli noktalardan biridir.

Page 39: MATEMATİK OLİMPİYATLARINA HAZIRLIK 1Herbirkitaptaöncelikle,konuya veokonuileilgiliörnek ö˘gretici olabilecek sorulara yerverdim.Dahasonra,herbirkonuileilgilidünyada de˘gi¸

Temel Bilgiler 39

F ve pozitif tamsayıları için,

=

esitliginde, ile aralarında asal

ve ile de aralarında asal ise, = ve = olmak zorundadır.

Örnek 65 n ∈ Z+ ve p, q asal sayılar ise, p (2n)2 = q6 (2n− 1) denkleminiçözünüz.Çözüm : Verilen denklemi

(2)2

2− 1 =6

seklinde yazalım. Sol taraftaki, 2 ve 2− 1 sayıları ardısık olduklarından aralarındaasaldır. Sag tarafta da ve asal oldugundan, sadelesemezler. O halde,

6 = (2)2 ve = (2− 1)

olmalıdır. 6 = (2)2 esitliginde, sag taraf çift ve asal oldugundan, sayısı bir çiftasal sayı yani 2 olmalıdır. Bu durumda, 26 = 42, esitliginden, = 4 olur. = 4

ise, = 7 olur.

Örnek 66 a, b ve c ∈ Z+ için, a+ 9b+ 20c = 50 denklemini saglayan kaç(a, b, c) üçlüsü vardır?Çözüm : ve sayıları pozitif tamsayı olduklarından, sayısı 3’ten küçük ol­malıdır. Yani, = 1 veya = 2 olabilir.

Eger, = 1 ise, +9 = 30 olur. Bu denklemde de, sayısı 1, 2 veya 3 olabilir.Eger, = 1 ise, = 21 , = 2 ise = 12 ve = 3 ise = 3 olur.Eger, = 2 ise, + 9 = 10 olur. Bu denklem ise sadece = 1 ve = 1

durumunda saglanır.Böylece, ( ) üçlüsü, (21 1 1) (12 2 1) (3 3 1) ve (1 1 2) olabilir. Yani,

4 tanedir. Yine, bu soruda da, pozitif tamsayılar yerine tamsayılar kümesi alınsaydı,sorunun sonsuz sayıda çözümü olurdu.

Örnek 671

x− 1

y=1

3denkleminin pozitif tamsayılarda kaç çözümü vardır?

Çözüm : Denklem düzenlenirse 3 − 3 = esitliginden,

=3

+ 3=3 + 9− 9

+ 3= 3− 9

+ 3

elde edilir. Buradan + 3 = 9 ve = 6 olabilir ki bu durumda = 2 elde edilir.Yani denklemin sadece 1 çözümü vardır ve bu çözüm (2 6)’dır.

Page 40: MATEMATİK OLİMPİYATLARINA HAZIRLIK 1Herbirkitaptaöncelikle,konuya veokonuileilgiliörnek ö˘gretici olabilecek sorulara yerverdim.Dahasonra,herbirkonuileilgilidünyada de˘gi¸

40 Matematik Olimpiyatlarına Hazırlık 1

Örnek 68 7 (x+ y) = xy denklemini saglayan kaç (x, y) tamsayı ikilisi vardır?Çözüm : 7 (+ ) = esitliginden, (− 7) ( − 7) = 49 olur. 49 sayısınıntamsayı bölenleri

−49 −7 −1 1 7 ve 49

oldugundan ( ) tamsayı çiftinin bir çözüm olabilmesi için ( )’nin(0 0) (14 14) (−42 6) (6−42) (8 56) (56 8)

ikililerinden biri olması gerekir. O halde 6 tane ( ) tamsayı ikilisi vardır.

Örnek 69 a ve b pozitif tamsayıları için, (a+ 2b) (a+ 4b) = 24 olacak sekildekaç (a, b) ikilisi vardır?Çözüm : (+ 2) (+ 4) ifadesinin çarpanlarının her ikisi de çift veya her ikisi detektir. Buna göre,

24 = 4 · 6 = 3 · 8 = 2 · 12 = 1 · 24oldugundan, + 4 + 2 oldugu da göz önüne alınırsa,½

+ 4 = 6

+ 2 = 4veya

½+ 4 = 12

+ 2 = 2

denklem sistemleri elde edilir. Fakat, ikinci denklem sistemini de dogrudan eleyebili­riz. Çünkü, ve pozitif tamsayılar oldugundan, +2 = 2 olması mümkün degildir.Buna göre, ilk denklem sisteminden, = 1 ve = 2 elde edilir. O halde bu denklemisaglayan tek pozitif tamsayı ikilisi ( ) = (2 1) olur.

F 2 − 2 = (− ) (+ ),F 3 − 3 = (− )

¡2 + + 2

¢,

F 3 + 3 = (+ )¡2 − + 2

¢.

Örnek 70 n ve n+ 100 sayılarının her ikisinin de pozitif bölenlerinin sayısı tekolacak sekilde kaç degisik n pozitif tamsayısı vardır?Çözüm : Bir sayının pozitif bölenlerinin sayısının tek olması için sayı tamkare ol­malıdır. O halde

= 2 ve + 100 = 2 + 100 = 2

olmalıdır. Buradan2 − 2 = ( − ) ( + ) = 100 = 2252

esitligine göre her iki çarpanın da çift olması gerektigi göz önüne alınırsa + = 50 ve − = 2

esitliklerinden = 26 ve = 24 elde edilir. O halde = 242 = 576 elde edilir.Yani istenen sekilde bir sayı vardır.

Page 41: MATEMATİK OLİMPİYATLARINA HAZIRLIK 1Herbirkitaptaöncelikle,konuya veokonuileilgiliörnek ö˘gretici olabilecek sorulara yerverdim.Dahasonra,herbirkonuileilgilidünyada de˘gi¸

Temel Bilgiler 41

Örnek 71 Kendisiyle, kendisinin 2 fazlasının toplamı 2’nin bir kuvvetine esit olan100’den küçük kaç pozitif tamsayı vardır?Çözüm : Sayının kendisi ise, 2 fazlası + 2 olur. Buna göre,

+ (+ 2) = 2 yani, 2 (+ 1) = 2

esitligi saglanmalı. Bu esitlige göre, + 1 sayısı da 2’nin bir kuvveti olmalıdır. Buise, = 1 3 7 15 31 63 sayıları için dogrudur.

Örnek 72 a2+b = b1999 denklemini saglayan kaç (a, b) tamsayı ikilisi vardır?(Estonya M.O. 1999)

Çözüm : 2 = ¡1998 − 1¢ yazalım. ≥ 2 için

ve 1998 − 1sayıları aralarında asal olacagından esitligin sol tarafının tamkare olması nedeniyle,bu sayıların her ikisi de tamkare olmalıdır. Fakat 1998 sayısı zaten bir tamkareoldugundan 1998 − 1 sayısı tamkare olamaz. Dolayısıyla ≥ 2 olamaz. Yani, ≤ 1 olmalıdır. Buna göre,

= 1 ise, = 0 = 0 ise, = 0 = −1 ise, = 0

olur. Ayrıca, ≤ −2 olamaz. Çünkü bu durumda da, 2 0 olur. O halde denkleminsaglayan ( ) pozitif tamsayı ikilileri,

(0 1) (0 0) ve (0−1)olmak üzere, 3 tanedir.

Örnek 73 1 + 2 + 3 + · · ·+ n ifadesi tamkare olacak sekilde 100’den küçüken büyük n pozitif tamsayısı kaçtır?Çözüm : ∈ Z olmak üzere, 1+2+3+· · ·+ = (+ 1)

2= 2 esitligini saglayan

100’den küçük en büyük pozitif tamsayısını arıyoruz. sayısı ile + 1 sayılarıaralarında asal oldugundan çarpımlarının tamkare olması için biri tamkare olmalı,digeri ise 2’ye bölündügünde tamkare olmalıdır. Buna göre, ve (+ 1) 2 tamkareolacak sekildeki sayıları arayalım. + 1 çift olması gerektiginden tek tamkarelerolmalıdır. 92 için (+ 1) 2 tamkare degildir. = 72 için, (+ 1) 2 = 25 olur ki,istenen kosulu saglar. Bu durumda, = 35 olur. Simdi de, + 1 ve 2 tamkareolacak sekildeki sayıları inceleyelim. 2 sayısının bir tamkare olması için, sayısı2’nin bir tek kuvveti olmalıdır. Bunu saglayan 100’den küçük tek sayı = 8’dir. Biz,istenen kosulu saglayan 100’den küçük en büyük tamsayısını aradıgımızdan yanıt = 49 bulunur.

Page 42: MATEMATİK OLİMPİYATLARINA HAZIRLIK 1Herbirkitaptaöncelikle,konuya veokonuileilgiliörnek ö˘gretici olabilecek sorulara yerverdim.Dahasonra,herbirkonuileilgilidünyada de˘gi¸

42 Matematik Olimpiyatlarına Hazırlık 1

Örnek 74 x ve y rasyonel sayılarının en sadelesmis durumlarında paydaları sıra­sıyla 60 ve 70’tir. Buna göre, x+ y sayısı, sadeslesmis durumda iken paydası enküçük kaç olabilir? (Harvard Math. Tourn. 2003)Çözüm : ve rasyonel sayılarının en sade halleri, sırasıyla 60 ve 70 olsun.Buna göre,

+ =

60+

70=7+ 6

420

ifadesinde, paydanın en küçük olması için, 7 + 6 ile 420’yi sadelestirebilecek enbüyük sayıyı bulmalıyız. sayısının 60 ile, sayısının da 70 ile aralarında asal olduguunutulmamalıdır. Buna göre, 7+ 6 sayısı kesinlikle 2, 3 ve 7 ile bölünemez. Bunubiraz daha açıklayalım.7 + 6 sayısında, 6 sayısı hem 2 hem de 3 ile bölünür. Fakat, 7 sayısı ne 2 ne de3 ile bölünemez. O halde, 7+ 6 sayısı asla 2 ve 3 ile sadelesemez. Benzer sekilde,7+ 6 sayısında, 7 sayısı 7 ile bölünürken, 6 sayısı asla 7 ile bölümez. Çünkü, ile 70 aralarında asal idi. Böylece, her ne olursa olsun, 7 + 6 sayısı, 2 3 ve 7 ilesadelesmeyecektir. Böylece, 420 = 22·3·5·7 oldugundan dolayı, 7+6 sayısı sadece5 ile sadelesebilecektir. = 1 ve = 3 alınması durumunda, 7+ 6 = 7+ 18 = 25olacaktır ve

1

60+3

70=5

84

esitliginden, yanıt 84 olarak bulunur.

Örnek 75 n2+4n− 7 ifadesi tamkare olacak sekilde kaç n tamsayısı vardır?Çözüm : ∈ Z için, 2 + 4− 7 = 2 olsun. Bu ifadeyi

2 + 4+ 4− 11 = 2

biçiminde yazalım. Buradan,(+ 2)

2 −2 = 11

olacaktır. Iki kare farkından,(+ 2−) (+ 2 +) = 11

elde edilir. Bu esitligin saglanması için,½−+ 2 = 1

++ 2 = 11,½

−+ 2 = 11

++ 2 = 1½−+ 2 = −1++ 2 = −11 ,

½−+ 2 = −11++ 2 = −1 ,

olmalıdır. Bu denklem sistemlerinden de, = 4 veya = −8 elde edilir.

Page 43: MATEMATİK OLİMPİYATLARINA HAZIRLIK 1Herbirkitaptaöncelikle,konuya veokonuileilgiliörnek ö˘gretici olabilecek sorulara yerverdim.Dahasonra,herbirkonuileilgilidünyada de˘gi¸

Temel Bilgiler 43

Örnek 76 x, y, z, w ∈ R olmak üzere,⎧⎪⎪⎨⎪⎪⎩2x+ y + z + w = 1

x+ 3y + z + w = 2

x+ y + 4z + w = 3

x+ y + z + 5w = 25

denklem sistemi veriliyor. w degeri kaçtır? (Harvard MIT Math. Tournament 2002)

Çözüm : Denklem sistemindeki denklemleri sırasıyla, 12 6 4 ve 3 ile çarpalım.Böylece, ⎧⎪⎪⎨⎪⎪⎩

24+ 12 + 12 + 12 = 12

6+ 18 + 6 + 6 = 12

4+ 4 + 16 + 4 = 12

3+ 3 + 3 + 15 = 75

elde edilir. Denklemleri taraf tarafa toplarsak,37+ 37 + 37 + 37 = 111

elde edilir. Buradan da, +++ = 3 bulunur. Simdi, dördüncü denklemde bunukullanırsak, 3 + 4 = 25 esitliginden, = 112 bulunur.

Örnek 77 q¡p2+p

¢= 15r − 3pr esitligini saglayan kaç (p, q, r) asal sayılar

üçlüsü vardır?

Çözüm : Verilen esitligi, ( + + 3) = 3 · 5 ·

seklinde yazabiliriz. = 3 ise, (3 + + 3) = 5 · ⇒ 4 = 2 olmalıdır. Bu esitligi saglayan ve

asal sayıları olamaz. Çünkü, her ikisi de çift olmalı ve birbirine esit olmamalı. = 5 ise, (5 + + 3) = 3 · ⇒ 6 = 0 oldugundan, = 0 olur ki, buradan daçözüm elde edilemez. = ise, ( + + 3) = 3 · 5 esitliginden,

=15− 3 + 1

=−3 − 3 + 18

+ 1= −3 + 18

+ 1

olması gerekir. Bu esitlige göre, +1 sayısı 18’in böleni olmalıdır. asal oldugundan,2 veya 17 olabilir.

= 2⇒ = 3 ve = 17⇒ = −2olur. O halde, tek çözüm ( ) = (2 3 2) bulunur.

Page 44: MATEMATİK OLİMPİYATLARINA HAZIRLIK 1Herbirkitaptaöncelikle,konuya veokonuileilgiliörnek ö˘gretici olabilecek sorulara yerverdim.Dahasonra,herbirkonuileilgilidünyada de˘gi¸

44 Matematik Olimpiyatlarına Hazırlık 1

1.8 Basit Esitsizlikler

Esitsizliklerle ilgili temel bilgilerimizi hatırlayalım.I 0 olması durumunda, tek ise, 0 ve çift ise, 0 olur.I Her ∈ R için, 2 ≥ 0’dır.⎧⎨⎩ 2 ≤ 1 ise, ∈ [−1 1] olmalıdır.

2 ≥ 2 ise, ≥ veya ≤ − olmalıdır.2 ≤ ise, ∈ [0 1] olmalıdır.

I olsun. Bu durumda,⎧⎪⎨⎪⎩ 0 ise ve

0 ise ve

olur.

I 0 ise 1

1

ve ( ∈ Z+) olur.

I 0 ise 1

1

ve ∈ Z+ için, tek ise, ve çift ise,

olur.I 2 + 2 + 2 = 0 esitlginin saglanması için = = = 0 olmalıdır.

Örnek 78 x2−y < −3 ve x2+y < 7 esitsizliklerini saglayan kaç (x, y) tam­sayı ikilisi vardır?Çözüm : Esitsizlikleri taraf tarafa toplarsak, 22 4 olur. Buna göre 0 ≤ 2 2

yani, ∈ 0 1−1 olur. Diger taraftan, 2 + 3 7 − 2 oldugundan, = 0oldugunda, ∈ 4 5 6 ve = ±1 oldugunda da = 5 olacagından, 5 tane ( )ikilisi vardır.

Örnek 79 a2 ≤ a ve b2 ≤ 1,1

c2≤ 1 olduguna göre, a+ b = c olacak sekilde

kaç tamsayısı vardır?Çözüm : Ilk esitsizlikten 0 ≤ ≤ 1 2 ≤ 1 esitsizliginden −1 ≤ ≤ 1 ve1

2≤ 1 esitsizliginden de 2 ≥ 1, yani ≥ 1 veya ≤ −1 esitsizlikleri elde edilir.

0 ≤ ≤ 1 ve −1 ≤ ≤ 1esitsizliklerinden, −1 ≤ + ≤ 2 olur. Buna göre, + ∈ −1 0 1 2 olur. ≥ 1veya ≤ −1 oldugundan, + = esitligini saglayan tamsayıları, −1 1 ve 2olabilir.

Page 45: MATEMATİK OLİMPİYATLARINA HAZIRLIK 1Herbirkitaptaöncelikle,konuya veokonuileilgiliörnek ö˘gretici olabilecek sorulara yerverdim.Dahasonra,herbirkonuileilgilidünyada de˘gi¸

Temel Bilgiler 45

Örnek 80 x, y pozitif tamsayılar olmak üzere,

2 <x

y< 3 ve

1

2<10

y<5

2

kosullarını saglayan (x, y) ikililerinin sayısı kaçtır?Çözüm : Ikinci esitsizligi düzenlersek,

2

102

5

olur. Buradan da, 4 20 elde edilir. Yani, ∈ 5 6 19 olur. Diger taraftan,2 3 esitsizliginde,

= 5 için, 10 15 ise, ∈ 11 12 13 14; = 6 için, 12 18 ise, ∈ 13 14 15 16 17; = 7 için, 14 21 ise, ∈ 15 16 17 18 19 20;

· · · = 19 için, 38 57 ise, ∈ 39 40 55 56 olacaktır. Yani,

sırasıyla, 4 5 18 farklı tamsayı degerleri alabilir. Buna göre, istenen kosulusaglayan

4 + 5 + 6 + · · ·+ 18 = 18 + 4

2(18− 4 + 1) = 165

( ) tamsayı ikilisi vardır.

Örnek 818

15<

n

n+ k<

7

13esitsizligini, sadece bir k tamsayısı için saglayan

en büyük n pozitif tamsayısı kaçtır? (AIME 1987)

Çözüm :8

15

+ 7

13esitsizliginin sol ve sag tarafından,

8+ 8 15 ve 13 7+ 7

olur. Bu iki esitsizlikten de,6

7

7

8

bulunur.7

8− 67=

56oldugundan, 112 oldugunda,

µ6

77

8

¶aralıgında

kesinlikle iki tamsayı olacaktır. = 112 alınırsa,6 · 1127

7 · 1128

esitsizliginden 96 98 elde edilir. Bu aralıkta, sadece bir tamsayısı vardır. Ohalde, en büyük pozitif tamsayısı 112’dir.

Page 46: MATEMATİK OLİMPİYATLARINA HAZIRLIK 1Herbirkitaptaöncelikle,konuya veokonuileilgiliörnek ö˘gretici olabilecek sorulara yerverdim.Dahasonra,herbirkonuileilgilidünyada de˘gi¸

46 Matematik Olimpiyatlarına Hazırlık 1

1.9 Faktöriyel Kavramı

1’den ’ye kadar olan sayıların çarpımını ! ile gösteririz. 0! = 1 olarak tanım­lanır. Örnegin, ≥ 3 için,

! = · (− 1) · (− 2) · (− 3)!seklinde yazılabilir. Asagıda, faktöriyel içeren denklemlerle bazı ilginç soruları ince­leyiniz.

Örnek 82 k ven pozitif tamsayılar olmak üzere, ((3!) !) ! = 3! · k · n! ise, k+ n

sayısının alabilecegi en küçük deger kaçtır? (AIME 2003)

Çözüm : ((3!)!)! = (6!)! = 720! = 720 · 719! = 3! · 120 · 719! oldugundan, = 120ve = 719’dur. O halde, + sayısının alabilecegi en küçük deger 120+719 = 839olur.

Örnek 8321!22!23! · · · 30!11!12!13! · · · 20! ifadesinin

a) bir rasyonel sayının karesi olması için,b) bir tamkare olması için,

en küçük hangi pozitif tamsayıyla çarpılması gerekir?

Çözüm :21!22! · · · 30!11!12! · · · 20! =

(21!)222 · (23!)2 24 · (25!)2 26 · (27!)2 28 · (29!)2 30

(11!)212 · (13!)2 14 · (15!)2 16 · (17!)2 18 · (19!)2 20

seklinde yazabiliriz. Paydadaki kareli ifadeler kesrin payındaki kareli ifadelerin içindeoldugundan dolayı sadelestiginde yine tamkare olacaktır. O halde, bu ifadede ki tümtamkareleri 2 ile gösterirsek,

= 222 · 24 · 26 · 28 · 3012 · 14 · 16 · 18 · 20 =

2 · 11 · 132 · 2 · 3

olur ki, olur ki, 2 bir çift sayı oldugundan dolayı, bu sayının hem bir rasyonel sayınınkaresi, hem de bir tamsayının karesi olması için 11 · 13 · 3 ile çarpılması yeterlidir.

Örnek 84 1! + 2! + 3! + · · ·+ 100! sayısının son rakamı kaçtır?Çözüm : 5!’den sonraki tüm faktöriyellerin son rakamı 0 oldugundan, sadece,

1! + 2! + 3! + 4!

toplamının son rakamını hesaplamak yeterlidir. 1+2+6+24 = 33 oldugundan, sonrakam 3 bulunur.

Page 47: MATEMATİK OLİMPİYATLARINA HAZIRLIK 1Herbirkitaptaöncelikle,konuya veokonuileilgiliörnek ö˘gretici olabilecek sorulara yerverdim.Dahasonra,herbirkonuileilgilidünyada de˘gi¸

Temel Bilgiler 47

Örnek 85 5 · 10 · 15 · 20 · · · 500 sayısınına) sagdan 97’inci rakamı kaçtır?b) sagdan 98’inci rakamı kaçtır?

Çözüm : a) Bu sayının sonunda kaç sıfır oldugunu hesaplayalım.5 · (2 · 5) · (3 · 5) · (4 · 5) · · · (100 · 5) = 5100 · (1 · 2 · 3 · · · 100)

= 5100 · 100!esitligine göre, yeterince 5 oldugundan 100! içinde kaç tane 2 çarpanı oldugunu bu­lalım.100 : 2 = 50 50 : 2 = 25 25 : 2 = 12 12 : 2 = 6 6 : 2 = 3 3 : 2 = 1

oldugundan,

50 + 25 + 12 + 6 + 3 + 1 = 97

tane 2 çarpanı vardır. O halde, 5100 · 100! sayısının sonunda 97 tane sıfır vardır. Bunagöre, 97’inci rakam 0 olur.b) Sayının tüm sıfırları atıılırsa, geriye 0 haricindeki sayılar ve 53’ün çarpımıyla eldeedilen bir sayı kalmıstır. 5 sayısının 0 haricindeki bir sayıyla çarpımı daima 5’in katıolacagından 98’inci rakamı 5 olacaktır.

Örnek 86 xy iki basamaklı bir sayı olduguna göre, x! + y! = xy denklemininkaç çözümü vardır?Çözüm : 5! = 120 oldugundan, ve rakamları 5’ten küçük olmalıdır. Bu özelligesahip tüm sayılardan, yani 10 11 12 44 sayılarından hiçbirinin verilen denklemisaglamadıgı görülebilir. Yani, denklemin çözümü yoktur.

Örnek 87 x, y ve z karısık olarak verilmis ardısık negatif olmayan tamsayılarolmak üzere,

x! = y! + z!

esitligini saglayan kaç (x, y, z) üçlüsü vardır?Çözüm : ’in bu ardısık sayıların en büyügü olacagı açıktır. = + 1 denilirse, ve , sırasıyla, − 1 ve olacaktır. Buna göre, sırasıyla,

(+ 1)! = ! + (− 1)!(+ 1) (− 1)! = (− 1)! + (− 1)!

2 + = + 1

olur. En son elde edilen 2 = 1 esitliginden = 1 elde edilir. O halde, ( ) (2 1 0) veya (2 0 1) olabilir.

Page 48: MATEMATİK OLİMPİYATLARINA HAZIRLIK 1Herbirkitaptaöncelikle,konuya veokonuileilgiliörnek ö˘gretici olabilecek sorulara yerverdim.Dahasonra,herbirkonuileilgilidünyada de˘gi¸

48 Matematik Olimpiyatlarına Hazırlık 1

Örnek 88 n! = a! + b! + c! denkleminin kaç çözümü vardır? (KANADA M.O1983)

Çözüm : Denkleme göre, , olacagı açıktır. Buna göre, ≤ − 1 vedolayısıyla da,

! + ! + ! ≤ 3 (− 1)!olacaktır. 3 için, ! 3 (− 1)! olacagından, 3 için çözüm yoktur (sayısının nasıl sınırlandıgını inceleyiniz.)

O halde, = 1 2 veya 3 olabilir. Kontrol edilirse, 3! = 2! + 2! + 2! oldugundan = 3 ’ün denklemin bir çözümü oldugu görülür.

Örnek 89 Her bir terimi 15’ten küçük veya esit olan a1, a2, ..., a pozitif tam­sayı dizisi için, a1! + a2! + · · ·+ a! ifadesinin son dört rakamının 2001 olmasıiçin, n en küçük kaç olmalıdır? (CENTRO Amerikan M.O 2001)

Çözüm : 15’ten küçük sayıların faktöriyellerinin son dört rakama kadar degerleriniyazalım.

1! 7→ 1 2! 7→ 2 3! 7→ 6 4! 7→ 24 5! 7→ 120, 6! 7→ 720,7! 7→ 5040, 8! 7→ 0 320 9! 7→ 2880, 10! 7→ 8800, 11! 7→ 6800,12! 7→ 1600, 13! 7→ 0 800, 14! 7→ 1200, 15! 7→ 8000

Faktöriyellerin son dört rakamının toplamının 2001 olmasını istiyoruz. Görüldügügibi tüm faktöriyellerde son rakamı tek olan sayı sadece 1 oldugundan 1 kesinlikleolmalıdır. Geriye 2000 toplamı kalır. Buldugumuz esitliklerden görülür ki, son dörtrakamın toplamı 2000 olabilecek sayılar sadece 13! ve 14! olabilir. O halde, enküçük 3 olur.Not : Bu soruda, terimlerin 15’ten küçük olması sartı kaldırılsaydı, cevap 2 olacaktı.Çünkü, 19! sayısının son dört rakamı 2000’dir ve 19! + 1! sayısının son dört rakamıda 2001 olur.

F Ardısık tane tamsayının çarpımı 1’den ’ye kadar ( dahil) tüm sayılar ile tambölünür. Hatta ! ile bölünür.

Örnek 90 n pozitif tamsayı olmak üzere n5−5n3+4n sayısının daima 120’yebölünecegini gösteriniz.Çözüm : Verilen ifadeyi çarpanlarına ayıralım.

5 − 53 + 4 = ¡4 − 52 + 4¢ =

¡2 − 1¢ ¡2 − 4¢

= (− 2) (− 1) (+ 1) (+ 2)olur. Yani 5 − 53 + 4 sayısı ardısık 5 sayının çarpımıdır ve 5! = 120’ye bölünür.

Page 49: MATEMATİK OLİMPİYATLARINA HAZIRLIK 1Herbirkitaptaöncelikle,konuya veokonuileilgiliörnek ö˘gretici olabilecek sorulara yerverdim.Dahasonra,herbirkonuileilgilidünyada de˘gi¸

Temel Bilgiler 49

Örnek 91 n ∈ Z+ olmak üzere, (n+ 10) (n+ 11) · · · (n+ 44) sayısının 315’ebölünebildigini ama, 316’ya bölünemeyebilecegini gösteriniz.Çözüm : 44−10+1 = 35 oldugundan bu çarpım daima 35!’e tam bölünecektir. 35!içinde 1 tane 27’ye, 3 tane 9’a, ve 11 tane 3’e bölünen çarpan vardır. Yani verilenifade kesinlikle 31+3+11 = 315’e bölünür. Diger yandan,

(+ 10) (+ 11) · · · (+ 44) = (+ 44)!

(+ 9)!

ifadesinde, paydadaki ifadeyi 27 olacak sekilde = 18 alırsak, 62!27! olur ki, 62!içinde, 2 + 6 + 20 = 28 tane 3 çarpanı, 27! içinde ise, 1 + 3 + 9 = 13 tane 3 çarpanıolacagından, 62!27! sayısının sadece 28− 13 = 15 tane 3 çarpanı bulunur. Yani, 316ya bölünmez.FWilson Teoremi :

bir asal sayı olmak üzere(− 1)! + 1 ≡ 0 (mod )

denkligi saglanır. Örnegin, 12! + 1 sayısı 13’e tam bölünür.

Örnek 92 a) 30! sayısının 31’e bölümünden kalan kaçtır?b) 11 · 12 · 12! sayısının 13’e bölümünden kalan kaçtır?c) 10! sayısının 13’e bölümünden kalan kaçtır?Çözüm : a) Wilson Teoremine göre,

30! ≡ −1 ≡ 30 (mod 31)olacagından, 30! sayısının 31’e bölümünden kalan 30’dur.b) Wilson Teoremine göre, 12! ≡ −1 (mod 13) oldugundan,

11 · 12 · 12! = 11 · 12 · (−1) ≡ 11 · (−1) (−1) ≡ 11 (mod 13)bulunur.c) Wilson Teoremine göre, 12! ≡ −1 (mod 13) oldugundan,

12 · 11 · 10! ≡ −1 (mod 13)yazılabilir. Buradan, (−1) · (−2) · 10! ≡ 12 (mod13) seklinde yazarsak,

10! = 6 (mod 13)

elde edilir.Alıstırmalar : a) 16! sayısının 17’ye bölümünden kalan kaçtır?b) 15 · 16! sayısının 17’ye bölümünden kalan kaçtır?c) 14! sayısının 17’ye bölümünden kalan kaçtır?Yanıt : a) 16, b) 2, c) 8.

Page 50: MATEMATİK OLİMPİYATLARINA HAZIRLIK 1Herbirkitaptaöncelikle,konuya veokonuileilgiliörnek ö˘gretici olabilecek sorulara yerverdim.Dahasonra,herbirkonuileilgilidünyada de˘gi¸

50 Matematik Olimpiyatlarına Hazırlık 1

1.10 Bir Sayının Tamkısmı (Tamdeger) ve Kesir Kısmı

Bir reel sayıdan büyük olmayan en büyük tamsayıya bu reel sayının tamdegeridenir. Bir reel sayısının tamdegeri db||ce ile, geri kalan virgüllü kısmı da ilegösterilir. Yani, = db||ce+ seklindedir. Buradaki, ifadesine de sayısınınkesir kısmı denir. = − db||ce seklinde bulunur. Örnegin,½ db|7 45|ce = 7

7 45 = 0 45

½ db||ce = 3 = 0 1415

½ §¥√3¯¦= 1©√

3ª= 0 732

½ db|−6 2|ce = −7−6 2 = 0 8

gibi. db||ce = ∈ ise, ≤ + 1 olması gerektigini görünüz.Tamdeger, sayılar teorisi bölümünde ayrıntılı olarak ele alınacaktır. Burada

sadece bazı sorularda gösterimin anlasılabilmesi için kısaca belirtilmistir.

Örnek 93 100! sayısı 3’ün en fazla kaçıncı kuvvetine bölünür?Çözüm : 1’den 100’e kadar olan sayılardan,§¥

1003

¯¦= 33 tanesi 3’e bölünür.§¥

10032

¯¦= 11 tanesi 32’ye bölünür.§¥

10033

¯¦= 3 tanesi 33’e bölünür§¥

10034

¯¦= 1 tanesi 34’e bölünür.

O halde, §¥10031

¯¦+§¥10032

¯¦+§¥10033

¯¦+§¥10034

¯¦= 33 + 11 + 3 + 1 = 48

oldugundan, 100! sayısı 3’ün en fazla 48’inci kuvvetine bölünür.

Örnek 94 x ve y pozitif tamsayılar olmak üzere,100!

50!= x7

ifadesinde y en büyük kaçtır?Çözüm :

§¥10071

¯¦+§¥10072

¯¦= 14+ 2 = 16 ve

§¥5071

¯¦+§¥5072

¯¦= 7+ 1 = 8 oldugundan,

100!

50!

ifadesinde en çok 16− 8 = 8 tane 7 çarpanı vardır ve en büyük 8 olur.

Page 51: MATEMATİK OLİMPİYATLARINA HAZIRLIK 1Herbirkitaptaöncelikle,konuya veokonuileilgiliörnek ö˘gretici olabilecek sorulara yerverdim.Dahasonra,herbirkonuileilgilidünyada de˘gi¸

Temel Bilgiler 51

Örnek 95 db|3x+ y|ce = 12 ve db|x+ 3y|ce = 14 ise db|x+ y|ce kaçtır?Çözüm : db|3+ |ce = 12 esitligine göre, 12 ≤ 3+ 13 olur.

db|+ 3|ce = 14 esitligine göre de 14 ≤ + 3 15 olur.Bu esitsizlikleri taraf tarafa toplarsak,

26 ≤ 4+ 4 28olur. Bu esitsizlige göre,

26

4≤ + 7

olacagından, db|+ |ce = 6 bulunur.

Örnek 96 x =§¥12x− 4

3

¯¦+ db|x|cedenklemini saglayan kaç x reel sayısı vardır?

Çözüm : Esitligin sag kısmında ifadelerin hepsi tamsayı oldugundan, sol kısmı olan sayısı da tamsayı olmalıdır. Bu durumda, db||ce = olacagından,

=§¥12− 4

3

¯¦+ ise,

§¥12− 4

3

¯¦= 0

olmalıdır. Son esitlikten de,

0 ≤ 12− 4

3 1

bulunur. Bu esitsizlikten8

3≤

14

3

çıkar. Bu aralıktaki tamsayıları = 3 ve = 4’tür.

Örnek 97§¥3−75

¯¦=x

3denklemini saglayan kaç x tamsayısı vardır?

Çözüm :§¥3−75

¯¦tamsayı oldugundan, esitligin sag tarafı olan

3de tamsayı olmalıdır

ve tamsayı oldugundan , 3’ün bir katı olmalıdır. Diger taraftan sırasıyla,

3≤ 3− 7

5

3+ 1

0 ≤ 3− 75−

3 1

0 ≤ 4− 2115

1

21

4≤ 9

esitsizlikleri elde edilir. Son esitsizligi de sadece = 6 tamsayısı saglar.

Page 52: MATEMATİK OLİMPİYATLARINA HAZIRLIK 1Herbirkitaptaöncelikle,konuya veokonuileilgiliörnek ö˘gretici olabilecek sorulara yerverdim.Dahasonra,herbirkonuileilgilidünyada de˘gi¸

52 Matematik Olimpiyatlarına Hazırlık 1

Örnek 98 x+ 4 = 2 x+3 db|x|ce− db|x|ce−13

olduguna göre x kaçtır?

Çözüm : Denklemde, = db||ce+ yazalım. Buna göre, sırasıyla

db||ce+ + 4 = 2 + 3 db||ce− db||ce− 13

3 db||ce+ 3 + 12 = 6 + 9 db||ce− db||ce+ 13 + 5 db||ce = 11

elde edilir. Bu esitlige göre, sag taraf tamsayı ve db||ce de tamsayı oldugundan,

∈ 0 1 2 olmak üzere, =

3olmalıdır. Buna göre,

+ 5 db||ce = 11esitliginden,

db||ce = 11−

5

olur. Buradan da, = 1 elde edilir. O halde, db||ce = 2 ve = 2 +1

3=7

3bulunur.

F = 0 esitligi ancak ve ancak bir tamsayıysa geçerlidir.

Örnek 99 db|4x|ce+ 4x− 5 = db|4− x|ce olduguna göre, x kaçtır?Çözüm : db|4|ce db|4− |ce ifadeleri tamsayı olduguna göre, 4 − 5 = 0 olmalıdır.Yani, 4 − 5 tamsayı olmalıdır. 4 − 5 = ∈ Z denilirse, =

+ 5

4olur. Bunu

denklemde yerine yazalım. Buna göre,db|+ 5|ce+ = §¥4− +5

4

¯¦denkleminde, + 5 tamsayı oldugundan, db|+ 5|ce = + 5’tir. O halde,

+ 5 = 4 +§¥−+5

4

¯¦esitliginden,

§¥−+54

¯¦= + 1 elde edilir. Buna göre, sırasıyla

+ 1 ≤ −+ 5

4 + 2

4+ 4 ≤ −− 5 4+ 8

4+ 9 ≤ − 4+ 13

9 ≤ −5 13

−95≥

−135

olacagından, = −2 elde edilir. Bu durumda, = + 5

4=3

4olur.

Page 53: MATEMATİK OLİMPİYATLARINA HAZIRLIK 1Herbirkitaptaöncelikle,konuya veokonuileilgiliörnek ö˘gretici olabilecek sorulara yerverdim.Dahasonra,herbirkonuileilgilidünyada de˘gi¸

Temel Bilgiler 53

Örnek 100 db|x|ce ve x ifadeleri, sırasıyla x gerçel sayısının tam ve kesirli kısım­larını göstermektedir. x ∈ R+ için, db|x|ce · x çarpımının 13’ten küçük olmamasıiçin, x sayısı en az kaç olmalıdır?

Çözüm : reel sayısı için, = db||ce+

ifadesinde, kesir kısmı 1’den küçük bir sayı olacagından, db||ce ile çarpıldıgında,db||ce degerinden daha küçük bir deger elde edilecektir. O halde, db||ce · çarpımının13’ten büyük veya esit olabilmesi için, db||ce sayısı en az 14 olmalıdır. Buna göre, enküçük degeri için, db||ce · çarpımını 13 alabiliriz. Bu durumda,

= 14 +13

14=209

14

sayısı, istenen kosulu saglayan en küçük reel sayısıdır.

Page 54: MATEMATİK OLİMPİYATLARINA HAZIRLIK 1Herbirkitaptaöncelikle,konuya veokonuileilgiliörnek ö˘gretici olabilecek sorulara yerverdim.Dahasonra,herbirkonuileilgilidünyada de˘gi¸

54 Matematik Olimpiyatlarına Hazırlık 1

1.11 Mutlak Deger

Bir tamsayının mutlak degeri, o sayının sayı dogrusunda bulundugu noktanınbaslangıç noktasına uzaklıgıdır. Bir sayısının mutlak degeri, || ile gösterilir.

|| =½

≥ 0 ise− 0 ise

seklinde yazılabilir. Mutlak degerin asagıdaki özelliklerini inceleyiniz.

1. |−| = ||2. ∈ R+ olmak üzere, || = ise = − veya = olur.3. ∈ R+ olmak üzere, || ise − olur.4. ∈ R+ olmak üzere, || ise − veya olur.5. || || = || 6. 6= 0 için,

|||| =

¯

¯

7. ∈ için, || = ||.

Örnek 101 |x− 5|+ |3− x| = 2 denkleminin kaç tamsayı çözümü vardır?Çözüm : Denklemi asagıdaki üç durum için inceleyecegiz.

i) ≤ 3 ii) 3 5 iii) 5 ≤

Bu üç duruma teker teker bakalım.i) ≤ 3 ise, −+ 5 + 3− = 2 esitliginden = 3 çözümü elde edilir.ii) 3 5 ise, − + 5 − 3 + = 2, yani 2 = 2 oldugundan, 3 5

aralıgındaki her reel sayı bir çözümdür.iii) 5 ≤ ise, − 5− 3 + = 2 esitliginden, = 5 çözümü elde edilir.

Denklemin tamsayı çözümlerini aradıgımızdan, = 3 = 5 ve = 4 çözüm olur.

Örnek 102|x− 2008|−100

|x2−2009x|+ |x− 2009| ≤ 0 esitsizligini saglayan kaç x tam­

sayısı vardır?Çözüm : = 2009 hariç,

¯2 − 2009

¯+ |− 2009| 0 oldugundan,

|− 2008|− 100 ≤ 0saglanmalıdır. = 2009 için payda sıfır olacagından, = 2009 olamayacagı unutul­mamalıdır. Buna göre,

|− 2008| ≤ 100esitsizliginden, −100 ≤ − 2008 ≤ 100 yani, 1908 ≤ ≤ 2108 olur. Bu aralıkta,2107 − 1907 + 1 = 201 tamsayı vardır. Fakat, 2009 saglamadıgından, 1 çıkarılırsa200 tamsayının bu esitsizligi sagladıgı görülür.

Page 55: MATEMATİK OLİMPİYATLARINA HAZIRLIK 1Herbirkitaptaöncelikle,konuya veokonuileilgiliörnek ö˘gretici olabilecek sorulara yerverdim.Dahasonra,herbirkonuileilgilidünyada de˘gi¸

Temel Bilgiler 55

Örnek 103 8 < |x− 3| < 13 esitsizligini saglayan kaç tane x tamsayısı vardır?Çözüm : 8 |− 3| 13 esitsizliginin saglanması için,

8 −+ 3 13 veya 8 − 3 13olmalıdır. Bu esitsizliklerden sırasıyla

−10 −5 veya 11 16

elde edilir. Böylece çözüm kümesi, : −10 −5 veya 11 16

bulunur. O halde esitsizligi saglayan tamsayılar,−9 −8 −7 −6 12 13 14 ve 15

sayılarıdır.

Örnek 104 4x+ 5y = 7 denklemini saglayan x, y tamsayıları için, 5 |x|−3 |y|ifadesinin en küçük degeri kaç olur?Çözüm : Denklemi saglayan tamsayıların farklı isaretli olacagı asikardır. Buna göre,

i) 0 ve 0 olursa,

5 ||− 3 || = −5− 3 = −5− 3µ7− 45

¶= −3− 4− 1− 2

5

olur. Bu ifade en küçük = −2 için, 1 degerini alır.ii) 0 ve 0 olursa,

5 ||− 3 || = 5+ 3 = 5+ 3µ7− 45

¶= 2+ 4 +

1 + 3

5

olur ve bu ifade en küçük = 3 için, 12 degerini alır. O halde en küçük degeri 1bulunur.

Örnek 105¯¯¯¯¯

x2+3x− 5¯−4¯−3¯−2¯−1¯= 3x− 15 denkleminin kaç

çözümü vardır?Çözüm : Esitligin sol tarafı pozitif oldugundan sag tarafı da pozitif olmalıdır. Bunagöre, 3− 15 0 esitsizliginden 5 olur. O halde, esitligin sol tarafında

2 + 3 25 + 15 = 40

oldugundan, mutlak degerlerin tümünün içi pozitif olacaktır. Yani sol taraftaki mutlakdegerlerin tümünü kaldırabiliriz. 2 + 3 − 15 = 3 − 15 esitliginden, 2 = 0,buradan da = 0 elde edilir, ki 5 olması gerektiginden, bu bir çözüm degildir.Yani denklemin kökü yoktur.

Page 56: MATEMATİK OLİMPİYATLARINA HAZIRLIK 1Herbirkitaptaöncelikle,konuya veokonuileilgiliörnek ö˘gretici olabilecek sorulara yerverdim.Dahasonra,herbirkonuileilgilidünyada de˘gi¸

56 Matematik Olimpiyatlarına Hazırlık 1

Örnek 106

½ |x+ y| = 1

|x|+ |y| = 1denklem sisteminin çözüm kümesini bulunuz.

Çözüm : Dört ayrı durum için denklemleri inceleyelim.i) ≤ 0 ve ≤ 0 olması durumunda, denklemler½ −− = 1

−− = 1

olacaktır. Bu denklem sistemi + = −1 denklemine denktir. Bu bir dogrunun denk­lemidir ve bu dogru üzerindeki her nokta verilen denklem sisteminin bir çözümüdürii) ≤ 0 ve ≥ 0 olması durumunda, denklemler,½ |+ | = 1

−+ = 1

olacaktır. Ikinci denklemde = 1 + degerini, birinci denklemde yazarsak,|+ 1 + | = 1

olur. Bu esitlikten de, 2+ 1 = −1 veya 2+ 1 = 1 elde edilir, yani( ) = (−1 0) veya ( ) = (0 1)

olmalıdır.iii) ≥ 0 ve ≤ 0 olması durumunda,

= 1 + ve | + 1 + | = 1olacak ve bu denklemin çözümünden de, 2 + 1 = −1 veya 2 + 1 = 1 yani,

( ) = (0−1) veya ( ) = (1 0)

elde edilecektir.iv ) ≥ 0 ve ≥ 0 olması durumunda, denklemler½

+ = 1

+ = 1

olacaktır. Bu sistemden de, + = 1 elde edilir.O halde, denklem sisteminin çözüm kümesi,

= ( ) : 0 ve + = 1 ; = ( ) : 0 ve + = −1 ; = (−1 0) (0 1) (0−1) (1 0)

olmak üzere, ∪ ∪ olarak bulunur.

F |− |+ |+ | = gibi, simetrik mutlak degerli denklemlerde, 0 bir çözümse−0’da bir çözümdür.Örnegin, |− 2|+ |+ 2| = 6 denklemini hem = 3 hem de = −3 saglar.

Page 57: MATEMATİK OLİMPİYATLARINA HAZIRLIK 1Herbirkitaptaöncelikle,konuya veokonuileilgiliörnek ö˘gretici olabilecek sorulara yerverdim.Dahasonra,herbirkonuileilgilidünyada de˘gi¸

Temel Bilgiler 57

Örnek 107 |x− 10|+|x− 9|+· · ·+|x− 1|+|x|+|x+1|+· · ·+|x+10|=cdenkleminin tek çözümü oldugunu biliyoruz. Buna göre, c sayısını bulunuz.Çözüm : 0 verilen denklemin bir kökü olsun, bu durumda−0 da denklemi sagladı­gından denklemin bir köküdür. Denklemin tek kökü olduguna göre, 0 = −0olmalıdır. Buradan, 20 = 0 esitliginden 0 = 0 olur. Bu degeri denklemde yazarsak,

= 10 + 9 + · · · 1 + 0 + 1 + · · ·+ 9 + 10= 2 (1 + · · ·+ 9 + 10) = 210 · 11

2= 110

bulunur.

Örnek 10818

|x− 1|+ |x− 5|+ |x− 13| ifadesinin en büyük degeri kaçtır?

Çözüm : Bu ifadenin en büyük degeri için, payda en küçük olmalıdır. Paydadakimutlak degerli ifadeleri 0 yapan degerleri sayı dogrusu üzerinde gösterelim.

A B C 1 5 13

Simdi bu sayı dogrusu üzerinde öyle bir noktası seçecegiz ki, payda bu degeri içinen küçük olsun. Seçecegimiz bu nokta, B haricinde bir nokta olursa, bulunan uzunluk[] uzunlugundan büyük olacaktır. O halde, seçmemiz gereken nokta B noktası,yani = 5 degeri olmalıdır. Böylece, en büyük deger,

18

4 + 0 + 8=3

2

bulunur.

F |+ | ≤ ||+ || esitsizligi saglanır. Bu esitsizlige üçgen esitsizligi de denir.Kanıt : Her ∈ R için,

≤ || ve 2 + 2 = ||2 + ||2

yazılabilir. Buna göre,2 + 2 + 2 ≤ ||2 + 2 ||+ ||2

esitsizliginden, (+ )2 ≤ (||+ ||)2 elde edilir. Her iki tarafın karekökü alınırsa,

|+ | ≤ ||+ ||oldugu görülür.

Page 58: MATEMATİK OLİMPİYATLARINA HAZIRLIK 1Herbirkitaptaöncelikle,konuya veokonuileilgiliörnek ö˘gretici olabilecek sorulara yerverdim.Dahasonra,herbirkonuileilgilidünyada de˘gi¸

58 Matematik Olimpiyatlarına Hazırlık 1

1.12 Üslü ve Köklü Sayılar

Üslü ve köklü sayılarla ilgili belli baslı tanımları verip, üslü ve köklü sayılarınözelliklerini kısaca hatırlayalım.

I ∈ N olmak üzere, = tanez |

· · · · · ifadesinde sayısına taban ve sayısınada üs veya kuvvet denir. 6= 0 için, 0 = 1 biçiminde tanımlanır. 00 sayısınıtanımlamayacagız. Bu ifadeyi tanımsız olarak kabul edecegiz.I ∈ N için,

i) () = · ii) · = + iii) · = ( · ), iv)

´esitlikleri saglanır.

I ∈ N için,

= − biçiminde tanımlanır. Buna göre, = 0 alınırsa,

− =1

esitligi elde edilir.

I = esitliginin saglanması için gerek ve yeter sart = olmasıdır.I ∈ Z+ olmak üzere,

√ sayısına, sayısının ’inci dereceden kökü denir.

sayısının çift sayı olması durumunda, √ ifadesinde negatif olamaz.

I ∈ Z+ ve ∈ Z ve ∈ R+ için, √ =

biçiminde tanımlanır. Böylece,

rasyonel üslü ifadeleri tanımlamıs oluruz. Tamsayı üslü sayılardaki özellikler, ras­yonel üslü sayılarda da geçerlidir.I ∈ Z+ ve ∈ Z için, (

√)

= √ esitligi saglanır.

I ∈ Z+ için, √ sayısının degeri sayısının tek ve çift olmasına göre degisir.

√ =

½ tek sayı ise,|| çift sayı ise

biçiminde tanımlanır.I ∈ Z+ olmak üzere,√ =

·√· =

√ =

√ ·

√=

r

ve

p√ = ·√

esitlikleri saglanır.I ∈ Z+ için, ise

√ √ biçiminde sıralanır.

I ∈ R için, 1 = 1 olarak tanımlanır. Ayrıca,

(−1) =½ −1 tek ise,

1 çift ise,biçiminde tanımlanır.I Buna göre, = 1 olması için gerek ve yeter kosul, i) 6= 0 ve ∈ R ve = 0, ii) = 1 ve ∈ R iii) = −1 ve çift tamsayı, durumlarından birininsaglanmasıdır.

Page 59: MATEMATİK OLİMPİYATLARINA HAZIRLIK 1Herbirkitaptaöncelikle,konuya veokonuileilgiliörnek ö˘gretici olabilecek sorulara yerverdim.Dahasonra,herbirkonuileilgilidünyada de˘gi¸

Temel Bilgiler 59

Paydayı Rasyonel Yapma

∈ R ve 6= 0 olmak üzere, biçiminde ifadeler bir rasyonel sayıolmak zorunda degildir. Bu nedenle bu tür ifadelere kesir demek dogru olmaz. Fakat,genel olarak sayısının köklü bir sayı olması istenmez. Bunun için, ifadesi ileçarpıldıgında bir rasyonel sayı elde edilecek sekilde, hem hem de sayısı uygunbir reel sayıyla çarpılabilir. Simdi, verilen bir sayısına göre, sayısını rasyonelyapacak uygun çarpanları verelim.

1. 6= 0 için,

seklindeki ifadeler √− ile çarpılır.

2. 2√− 2

seklindeki ifadeler 2√+ 2

√ ile çarpılır.

3. 2√+ 2

seklindeki ifadeler 2√− 2

√ ile çarpılır.

4.

3√− 3√

seklindeki ifadeler 3√2 +

3√+

3√2 ile çarpılır.

5. 3√+ 3√

seklindeki ifadeler 3√2 − 3

√+

3√2 ile çarpılır.

Örnek 109 (x− 3)22−18 = 1 denkleminin kaç kökü vardır?

Çözüm : i) − 3 = 1 ve 22 − 18 ∈ R esitliginden = 4 olur.ii) − 3 6= 0 ve 22 − 18 = 0 ise 2 (− 3) (+ 3) = 0 esitliginden = 3

veya = −3 olur. Fakat 6= 3 oldugundan = −3 olabilir.iii) − 3 = −1 ve 22 − 18 çift olabilir. = 2 olur ve = 2 için, 22 − 18

sayısı çift oldugundan = 2 de bir çözümdür. O halde 3 çözüm vardır.

Örnek 110 5 · 22−36 · 2+32 −2 · 2 ·3+81 = 0 olduguna göre x ve yarasında daha basit bir bagıntı bulunuz.

Çözüm : 5 · 22 − 36 · 2 + 32 − 2 · 2 · 3 + 81 = 0 esitligini,¡22 − 2 · 2 · 3 + 32¢+ ¡22(+1) − 36 · 2 + 81¢ = 0

seklinde yazarsak,

(2 − 3)2 + ¡2+1 − 9¢2 = 0bulunur. Buradan, 2 = 3 ve 2+1 = 9 elde edilir. Sonuncu esitliklerin ikincisindenelde edilen 3 = 2(+1)2 ifadesi birinci esitlikte yazılırsa, 2 = 2(+1)2 esitliginden = 2 (+ 1) elde edilir.

Page 60: MATEMATİK OLİMPİYATLARINA HAZIRLIK 1Herbirkitaptaöncelikle,konuya veokonuileilgiliörnek ö˘gretici olabilecek sorulara yerverdim.Dahasonra,herbirkonuileilgilidünyada de˘gi¸

60 Matematik Olimpiyatlarına Hazırlık 1

Örnek 111√2x+ 3+ 4

√3− x+ 3

√+ 2 ifadesi bir reel sayı olacak sekilde kaç

x tamsayısı vardır?Çözüm : 2+ 3 ≥ 0 ve 3− ≥ 0 olmalıdır. Buna göre,

−32 ≤ ≤ 3olacagından, bu aralıktaki, tamsayılar −1 0 1 2 ve 3 olur.

Örnek 112 a, b ve c rasyonel sayılar olmak üzere,2

1−√5−√6 =a√30−√5

b− 3

c

esitligi saglanıyor ise a, b ve c’yi bulunuz.

Çözüm :2

1−√5−√6((1−

√5)+√6)

=√30

(√30)

−√5

− 3

esitliginin sol tarafını

¡1−√5¢ +√6

ile çarpalım. Buradan, sırasıyla,2¡1−√5 +√6¢−2√5 =

√30

30−√5

− 3

1−√5 +√6 =

5

− √6

6+3√5

elde edilir. Son esitlige göre, 5,√6 ve√5 sayıları birbirinden bagımsız olduklarından,

5

= 1 −1 = 3

ve 1 = −

6

olacagından, = 5 = −3 ve = −6 bulunur.

F Iç Içe Köklerp+ 2

√ veya

p− 2√ seklindeki ifadelerde, kök içerisindeki ifadenin bir

reel sayının karesi olabilmesi için, = + ve = ·

olacak sekilde ve sayılarının olması yeterlidir. Bu sekilde ve sayıları var ise,p+ 2

↓ ↓+ ·

=

r³√+√´2=√+√

p− 2√↓ ↓

+ ·

=

r³√−√´2=¯√

−√¯

seklinde dısarı çıkacaktır.

Page 61: MATEMATİK OLİMPİYATLARINA HAZIRLIK 1Herbirkitaptaöncelikle,konuya veokonuileilgiliörnek ö˘gretici olabilecek sorulara yerverdim.Dahasonra,herbirkonuileilgilidünyada de˘gi¸

Temel Bilgiler 61

Örnek 113p4−√7−

p4+√7 =?

Çözüm : Önce iç içe kök olan bu ifadeleri kuralımızı uygulayabilecek sekile sokmayaçalısalım. Bunun için, kökün içindeki ifadeyi 2 ile çarpıp bölüyoruz.p

4−√7 =r8− 2√7

2=

1√2(p8− 2√7)

esitliginde,p8− 2√7 =

¯√7−√1

¯oldugundan,p

4−√7 =√7−√1√2

olur. Benzer düsünceyle,p4 +√7 =

√7 +√1√

2bulunur. Böylece,

p4−√7−

p4 +√7 =

√7− 1√2−√7 + 1√2

=−2√2= −√2

elde edilir.

Örnek 114 0 < x < 1 olmak üzere,qx2+2

√x3+x−

qx2−2

√x3+x = 5

denkleminin kaç reel kökü vardır?

Çözüm : (p2 + + 2

√3

↓ ↓2 + 2 ·

) =√2 +

√ ve

p2 − 2

√3 + =

¯√2 −√

¯

esitliklerinde, 0 1 iken, 2 oldugundan,¯√

2 −√¯=√ − olur.

Böylece, p2 + 2

√3 + −

p2 − 2

√3 + = 5

denkleminde bu esitlikleri kullanırsak,+√−√+ = 5

olur. Bu denklemden de, 2 = 5 ve = 52 olur. Fakat, 0 1 oldugundan, bumümkün degildir. O halde denklemin kökü yoktur.

Örnek 115¡52 + 6

√43¢32− ¡52− 6√43¢32 =? (AIME 1990)

Çözüm :p52± 6√43 =

p52± 2√43 · 9 = √43±√9 = √43± 3 oldugundan,³

52 + 6√43´32

−³52− 6

√43´32

=³√43 + 3

´3−³√43− 3

´3esitliginde,

√43 + 3 = ve

√43− 3 = diyelim.

Page 62: MATEMATİK OLİMPİYATLARINA HAZIRLIK 1Herbirkitaptaöncelikle,konuya veokonuileilgiliörnek ö˘gretici olabilecek sorulara yerverdim.Dahasonra,herbirkonuileilgilidünyada de˘gi¸

62 Matematik Olimpiyatlarına Hazırlık 1

Bu durumda,³√43 + 3

´3−³√43− 3

´3= 3 − 3 = (− )

¡2 + + 2

¢esitliginde ve yerine yazılırsa,³√

43 + 3´3−³√43− 3

´3= 6

³43 + 6

√43 + 9 + 43− 9 + 43− 6

√43 + 9

´esitliginden

¡√43 + 3

¢3 − ¡√43− 3¢3 = 828 elde edilir.

Örnek 1162+√3√

2 +p2+√3+

2−√3√2−

p2−√3

=?

Çözüm :p2±√3 =

p4± 2√3√2

=

√3± 1√2

oldugundan,

2 +√3√

2 +p2 +√3+

2−√3√2−

p2−√3

=2 +√3

√2 +

√3 + 1√2

+2−√3

√2−√3− 1√2

=2√2 +√6

3 +√3

+2√2−√63−√3 =

√2

Ã2 +√3

3 +√3+2−√33−√3

!

=√2(2 +√3

3 +√3

(3−√3)

+2−√33−√3(3+√3)

) =√2

Ã3 +√3 + 3−√39− 3

!=√2

bulunur.

Örnek 117√3−√2 ve

31

100sayılarından hangisi daha büyüktür?

Çözüm :√3−√2 31

100oldugunu kabul edelim. Her iki tarafın karesini alırsak,

5− 2√6 961

104

yani 5− 961

10000 2√6 olur. Sol taraf 49 039

104oldugundan,

49 · 103104

2√6

yazılabilir. Tekrar kare alırsak,2401

102 24

esitsizligi elde edilir ki, bu yanlıstır. O halde,√3 −√2 31

100kabulümüz yanlıstır.

Yani,√3−√2 31100 olur.

Page 63: MATEMATİK OLİMPİYATLARINA HAZIRLIK 1Herbirkitaptaöncelikle,konuya veokonuileilgiliörnek ö˘gretici olabilecek sorulara yerverdim.Dahasonra,herbirkonuileilgilidünyada de˘gi¸

Temel Bilgiler 63

Örnek 118 2, 3, 4, ..., 1000 sayılarından kaçı küpkökünden küçük olan en büyüktamsayıya bölünür?

Çözüm :3√2 3√3 3

√8 sayılarının küpkökünden küçük olan en büyük tamsayı 1,

3√9 3√10 3

√27 sayılarının küpkökünden küçük olan en büyük tamsayı 2,

3√28 3√29 3

√64 sayılarının küpkökünden küçük olan en büyük tamsayı 3,

...3√513 3√514 3

√729 sayılarının küpkökünden küçük olan en büyük tamsayı 8,

3√730 3√731 3

√1000 sayılarının küpkökünden küçük olan en büyük tamsayı 9’dur.

En genel halde,

3 + 1 3 + 2 (+ 1)3

sayılarının köp kökünden küçük olan en büyük sayı ’dir. Burada,(+ 1)

3 − ¡3 + 1¢+ 1 = 32 + 3+ 1sayı oldugundan, bu 32 + 3+ 1 sayıdan, 1 için ile bölünenler :

3 + 3 + 2 3 +¡32 + 3

¢sayılarıdır. = 1 ise bu sayıların tamamı 1’e bölüneceginden kosulu saglarlar vebunların sayısı 7 tanedir. Fakat, 1 için, ile bölünenlerin sayısı, artıs miktarı oldugundan,

Son Terim − Ilk TerimArtıs Miktarı

+ 1 =3 +

¡32 + 3

¢− ¡3 + ¢

+ 1

= 3+ 3

tanedir. Buna göre, = 1 için 7 ve = 2 3 9 için 3 + 3 degerlerini toplarsakistenen elde edilir.

1 + (3·1 + 3) + (3·2 + 3) + (3·3 + 3) + · · ·+ (3·9 + 3)toplamını,

1 + 3 (1 + 2 + · · ·+ 9) + (3 + 3 + · · ·+ 3)seklinde yazarsak,

1 + 3 · 9 · 102

+ 27 = 163

buluruz.

Page 64: MATEMATİK OLİMPİYATLARINA HAZIRLIK 1Herbirkitaptaöncelikle,konuya veokonuileilgiliörnek ö˘gretici olabilecek sorulara yerverdim.Dahasonra,herbirkonuileilgilidünyada de˘gi¸

64 Matematik Olimpiyatlarına Hazırlık 1

1.13 Oran ve Orantı

En az biri sıfırdan farklı olan aynı birimdeki iki çoklugun bölümüne oran denir.Iki veya daha fazla oranın esitligine de orantı denilir.

=

=

orantısında sayısına orantı sabiti denir. Orantının bazı özelliklerini hatırlayalım.

*

=

=

orantısı : : = : : seklinde gösterilir.

* Bir orantıda, payların toplamı paydaların toplamına oranlandıgında orantı sabitidegismez.Eger

=

=

= ise, + +

+ + = olur.

Eger

=

=

= ise,

+ +

+ + = olur.

* Eger =

=

ise, +

+ =

+

+ =

+

+ olur.

* Iki çokluktan biri artarken digeri de artıyorsa bu iki çokluk dogru orantılı, biriartarken digeri azalıyorsa bu iki çokluk ters orantılıdır denir. ve dogruorantılı ise,

= ve ters orantılı ise, = olur.

Örnek 1195x− y

x− 7y + 4=

2a

5a− 3b vex+ 3y

x− 7y + 4=

2b

5a− 3b oldugunagöre, x kaçtır?Çözüm : Birinci denklem 5 ile, ikinci denklem de−3 ile çarpılıp taraf tarafa toplanırsa,

25− 5 − 3− 9− 7 + 4 =

10− 65− 3 = 2

esitliginden, 25− 5 − 3− 9 = 2− 14 + 8 olur. Bu denklemin çözümünden, = 25 elde edilir.

Örnek 120 Sıfırdan ve birbirinden farklı a, b ve c sayıları için,a+ b

c=b+ c

a=c+ a

b= k

ise, k kaçtır?

Çözüm :+

+ 1 =

+

+ 1 =

+

+ 1 = + 1 esitliklerinden,

+ +

=

+ +

=

+ +

= + 1

olur. ve birbirinden farklı oldugundan bu esitligin saglanması için + + = 0

olmalıdır. Bu durumda, = −1 bulunur.

Page 65: MATEMATİK OLİMPİYATLARINA HAZIRLIK 1Herbirkitaptaöncelikle,konuya veokonuileilgiliörnek ö˘gretici olabilecek sorulara yerverdim.Dahasonra,herbirkonuileilgilidünyada de˘gi¸

Temel Bilgiler 65

1.14 Karısık Örnekler

Örnek 121 2, 56, 2, 61, 2, 65, 2, 71, 2, 79, 2, 82, 2, 86 sayılarınınher birini bir tamsayı degerine yuvarlanarak yapılan toplama islemindeki toplam,gerçek toplama esit olsun. Her bir yuvarlamadaki hataların en büyük olanı Eolsun. E sayısının en küçük degeri için, 100E sayısı kaçtır? (AIME 1985)

Çözüm : 7 sayının toplamı hesaplanırsa,2 56 + 2 61 + 2 65 + 2 71 + 2 79 + 2 82 + 2 86 = 19

bulunur. Açıktır ki, yuvarlamadan sonra yapılan toplamanın en küçük olması için,sayıların 2 ya da 3’e yuvarlanması gerekir. Sayıların 2 ve 3’e yuvarlanmasında hata 1 olurken, aksi halde 1 olacaktır. Böylece, bes tane 3 ve iki tane 2 olmasıgerektigi görülür. Yuvarlamalardaki en küçük hata yapılması için verilen sayılardanen küçük olan ikisini 2 için düsünmeliyiz. Böylece, yuvarlamalarda yapılan hatalarsırasıyla,

0 56, 0 61, 0 35, 0 29, 0 21, 0 18, 0 14

olacaktır. Bu hataların en büyügü = 0 61 ve 100 = 61 elde edilir.

Örnek 122 |x+ |x|+a|+ |x− |x|−a| = 2 denkleminin tam üç çözümü olacaksekilde kaç a sayısı vardır?Çözüm : denklemin bir çözümü olsun. Denklemde (−) yazalım.|(−) + |(−)|+ |+ |(−)− |(−)|− | = |− ||− |+ |+ ||+ | = 2

oldugundan, (−) de bir çözümdür. Denklemin üç tane, yani tek sayıda çözümüolması isteniyor. Bunun için, = 0 denklemin köklerinden biri olmalıdır. Denklemde = 0 yazarsak,

||+ |−| = 2

2 || = 2

esitligine göre = ±1 olmalıdır. = 1 ise, ≥ 0 için, 2+2 = 2 denkleminden, = 0 ve çözüm iken (−)

de bir çözüm oldugundan sadece 1 çözümü oldugu görülür. O halde, = 1 olamaz. = −1 ise, ≥ 0 ise, |2− 1| + 1 = 2 ise |2− 1| = 1 olur. Bu esitlikten

de, ½2− 1 = 12− 1 = −1

olmalıdır. Yani, = 0 ve = 1 olur. (−) yani, −1 de denklemin bir çözümüolacagından, = −1 için denklemin 3 çözümü oldugu görülür.

Page 66: MATEMATİK OLİMPİYATLARINA HAZIRLIK 1Herbirkitaptaöncelikle,konuya veokonuileilgiliörnek ö˘gretici olabilecek sorulara yerverdim.Dahasonra,herbirkonuileilgilidünyada de˘gi¸

66 Matematik Olimpiyatlarına Hazırlık 1

Örnek 123 x = 32007 olmak üzere,√x2+2x+ 4 ve

p4x2+2x+ 1 sayıları

arasında kaç tamsayı vardır?Çözüm : + 1

√2 + 2+ 4 + 2 ve 2

√42 + 2+ 1 2+ 1

oldugundan,√2 + 2+ 4 ve

√42 + 2+ 1 sayıları arasındaki tamsayı sayısı

2− (+ 2) + 1 = − 1bulunur.

Örnek 124 1 ≤ a ≤ 100 ve 1 ≤ b ≤ 100 olmak üzere, a+√b +

1

a+√b

ifadesi tamsayı olacak sekilde, kaç (a, b) tamsayı çifti vardır?Çözüm : Verilen ifadeyi asagıdaki gibi düzenleyelim.

= +√+

1

+√= +

√+ + 1

+√

= +

³√+

´− 2 + + 1

+√

= 2++ 1− 2

+√

Bu esitlige göre, 2 = + 1 ise ifadesi tamsayı olur. 2 6= + 1 oldugunda

tamsayı olamaz, çünkü, bu durumda+ 1− 2

+√= ∈ Z

olması gerekir. Buradan,

³+√´= + 1− 2

esitligine göre,√ =

+ 1− 2 −

olacagından son esitligin sag tarafı rasyonel sayı oldugundan, sol taraftaki√ sayısı

da bir rasyonel sayı olmalıdır. Bu durumda tamkare olmalıdır. Yani, +√ tamsayı

olur. Fakat, +√ ≥ 2 oldugundan, 1

+√

ifadesi ve dolayısıyla da

+√+

1

+√

ifadesi tamsayı olamaz. O halde, sadece 2 = + 1 olması durumunda ifadesitamsayı olabilir. = 2 − 1 esitligine göre,

∈ 2 3 10olabileceginden, istenen sekilde 9 sayı vardır.

Page 67: MATEMATİK OLİMPİYATLARINA HAZIRLIK 1Herbirkitaptaöncelikle,konuya veokonuileilgiliörnek ö˘gretici olabilecek sorulara yerverdim.Dahasonra,herbirkonuileilgilidünyada de˘gi¸

Temel Bilgiler 67

Örnek 125 6! = 8 · 9 · 10 esitliginde 6! sayısı ardısık üç sayının çarpımı sek­linde yazılabilmektedir. n! sayısı (n− 3) tane ardısık sayının çarpımına esit olacaksekilde yazılabiliyorsa, en büyük n sayısı kaç olabilir? (AIME 1990)

Çözüm : (− 3) tane ardısık sayının çarpımı bir pozitif tamsayı olmak üzere,(− 3 +)!

!

seklinde yazılabilir. Buna göre,

! =(− 3 +)!

!

esitliginin saglanması için, 3 olmalıdır. Burada, (− 3 +)! ! olacagın­dan,

! (+ 1) (+ 2) · · · (− 3 +)

!= !

yazılabilir. Bu esitlige göre,(+ 1) (+ 2) · · · (− 3 +) = !

olmalıdır. = 4 alınırsa, + 1 = 4! = 24 denklemine göre = 23 elde edilir. Biz

en büyük sayısını arıyoruz., bu nedenle ’nin daha büyük degeri için bu esitliginsaglanıp saglanmadıgını görelim.

= 5 alınırsa, (+ 1) (+ 2) = 120 esitliginden, 10 11 olur. = 6 alınırsa, (+ 1) (+ 2) (+ 3) = 6! esitliginden de 7 8 olur.

Yani, ’nin yaklasık degeri −3√! olacak ve arttıkça küçülecektir. O halde

istenen sekildeki en büyük tamsayısı 23 bulunur.

Örnek 126 Aralarındaki fark 60 olan iki pozitif tamsayının karekökleri toplamıtamkare olmayan bir pozitif tamsayının kareköküne esit olduguna göre, bu iki tam­sayının toplamının alacagı maksimum deger kaçtır? (AIME 2003)

Çözüm : Sayılarımız ve 60 + olsun.√ +√+ 60 =

√ esitliginin her iki

tarafının karesi alınırsa,2+ 60 + 2

√2 + 60 =

olur. Bu esitlige göre,√2 + 60 bir tamsayı olmalıdır. O halde, bir ∈ Z için,

2 + 60 = 2 yazılabilir. Buradan,2 + 60+ 900− 2 = 900

(+ 30)2 − 2 = 900

(+ 30− ) (+ 30 + ) = 900

olur.

Page 68: MATEMATİK OLİMPİYATLARINA HAZIRLIK 1Herbirkitaptaöncelikle,konuya veokonuileilgiliörnek ö˘gretici olabilecek sorulara yerverdim.Dahasonra,herbirkonuileilgilidünyada de˘gi¸

68 Matematik Olimpiyatlarına Hazırlık 1

Son esitlikte, her iki çarpımda çift olmalıdır. Soruda bu sayıların toplamının yani,2+60’ın alabilecegi en büyük deger soruluyor. Bunun için en büyük seçilmeli, bunedenle + 30 + çarpanını maksimum seçmeliyiz.

+ 30 + = 450 ve + 30− = 2

seçilirse, 2 + 60 = 452 denkleminden = 196 bulunur. Fakat bu durumda birtamkare oldugundan istenen sekilde degildir.

+ 30 + = 150 ve + 30− = 6

seçilirse, 2+ 60 = 156 denkleminden = 48 olur ve tamkare degildir. O halde buiki tamsayının toplamının alacagı maksimum deger 156 olur.

Örnek 127 2+1 = m+ 2n (m− 1) esitligini saglayan kaç (k,m, n) pozitiftamsayı üçlüsü vardır?Çözüm : + 2 (− 1) = (− 1) (2+ 1) + 1 seklinde yazalım. Bu du­rumda, 2 = (− 1) (2+ 1) olur. Bu yazılısta, (2+ 1) sayısının bir tek sayıoldugu göz önüne alınırsa, pozitif tamsayı olmak üzere 2 + 1 formundaki birsayıyı (− 1) (2+ 1) + 1 seklinde yazmak mümkün degildir. Çünkü, 2 + 1 =(− 1) (2+ 1) + 1 esitliginden,

2 = (− 1) (2+ 1)olur. Sol taraf 2’nin kuvveti iken sag tarafta bir tek çarpan olamaz. O halde, budenklemin çözümü yoktur.

Örnek 128 x ve y pozitif tamsayılar olmak üzere, x2+y ve x+ y2 sayılarınınher ikisinin birden tamkare olamayacagını gösteriniz. (SSCB M.O. 1966)

Çözüm : 2 sayısından büyük olan en küçük tamkare (+ 1)2 = 2 + 2 + 1

oldugundan, 2 + tamkare ise, 2 olmalıdır. Benzer sekilde, 2 sayısındanbüyük olan en küçük tamkare ( + 1)2 = 2+2+1 oldugundan, +2 tamkare ise 2 olmalıdır. Fakat, 2 ve 2 esitsizliklerinin aynı anda saglanmasıimkansızdır. O halde 2 + ve + 2 sayılarının her ikisi de tamkare olamaz.

Örnek 129 x, y ∈ R , y ≥ 0 ve y (y + 1) ≤ (x+ 1)2 olduguna göre,

y (y − 1) ≤ x2oldugunu gösteriniz.Çözüm : Ispatımızı, 0 ≤ ≤ 1 ve 1 için ayrı ayrı yapalım.i) 0 ≤ ≤ 1 olursa, ( − 1) ≤ 0 ≤ 2 oldugundan esitsizlik saglanır.

Page 69: MATEMATİK OLİMPİYATLARINA HAZIRLIK 1Herbirkitaptaöncelikle,konuya veokonuileilgiliörnek ö˘gretici olabilecek sorulara yerverdim.Dahasonra,herbirkonuileilgilidünyada de˘gi¸

Temel Bilgiler 69

ii) 1 olması durumundaysa, +1

2 veya +

1

2≤ olmasına göre

inceleyelim.

+1

2 ise,

− 12 − 1 ve

µ− 1

2

¶µ+

1

2

¶ ( − 1)

olacagından,

( − 1) ≤ 2 − 14 2

elde edilir. Benzer sekilde,

+1

2≤ ise,

( − 1) = ( + 1)− 2 ≤ (+ 1)2 − 2µ+

1

2

¶= 2

bulunur.

Örnek 130³2

3√2+1−

p12

3√2−15

´3= 32 oldugunu ispatlayınız.

Çözüm : 3√2 = diyelim, 3

√32 =

3√25 = 2

3√22 = 22 olacagından, denklemde

yerine yazılırsa, 2+ 1−√12− 15 = 22 olur. Kökü yalnız bırakırsak,√12− 15 = −22 + 2+ 1

olur. Bu esitligin dogrulugunu gösterirsek ispat biter.Her iki tarafın karesini alırsak,

12− 15 = ¡−22 + 2+ 1¢2 = 4− 83 + 44 + 1olur. Bu denklemde, 3 = 2 ve 4 = 2 oldugu kullanılırsa,

12− 15 = 4− 16 + 8+ 1 = 12− 15esitligi elde edilir. Yani, esitlik dogrudur.

Örnek 131 p bir asal sayı vex > 0n ≥ 0 tamsayılar olmak üzere,n2·p < 1000

esitsizligini saglıyorsa,

n2+100· xp= (n+ x)

2

denkleminin kaç (x, n, p) çözüm üçlüsü vardır? (AÜMO 2009)Çözüm : 2 · + 100 = 2 · + 2+ 2 esitliginden 100 = · (2+ ) eldeedilir. Bu denkleme göre, asal sayısının 2 veya 5 olabilecegini görürüz. Bu degerleriçin denklemi inceleyelim.

Page 70: MATEMATİK OLİMPİYATLARINA HAZIRLIK 1Herbirkitaptaöncelikle,konuya veokonuileilgiliörnek ö˘gretici olabilecek sorulara yerverdim.Dahasonra,herbirkonuileilgilidünyada de˘gi¸

70 Matematik Olimpiyatlarına Hazırlık 1

= 2 için, 2 + = 50 denkleminin çözümlerini bulalım. 22 1000 ise,2 500 ve buradan da ≤ 22 olacagından, istenen sekilde 23 tane negatif olmayan tamsayısı ve dolayısıyla da 23 tane ( ) ikilisinin oldugu görülür.

= 5 için, 20 = 2+ denklemi elde edilir. Buradan 0 kosulundan dolayı, ≤ 9 olması gerekir. 0 ≤ ≤ 9 sayıları, problemin 2 · 5 1000 yani, 2 200

kosulunu da saglarlar. O halde, = 5 durumunda da 10 tane ( ) ikilisi bulunur.Sonuç olarak, tüm çözümler sayısı 23 + 10 = 33 olur.

Örnek 132 n bir pozitif tamsayı olmak üzere, a =n (n+ 1)

2biçimindeki sayıya

bir üçgensel sayı denir. Buna göre, a− b = 90 esitligini saglayan kaç tane (a, b)üçgensel sayı ikilisi vardır? (AÜMO 2009)

Çözüm : = (+ 1)

2ve =

(+ 1)

2 ( ∈ N) esitliklerinden,

90 = − =1

2

¡2 + −2 −

¢90 =

1

2(−) (++ 1)

1 · 2 · 2 · 3 · 3 · 5 = (−) (++ 1)

olur. − ve ++1 sayılarının biri çift oldugunda digeri tek olacagından, küçükçarpan olan − sayısı 1 3 5 9 15 ve 4 degerlerini alabilir. − = 1 olursa,++1 = 180; − = 4 olursa, ++1 = 45 v.b. Buradan ve dolayısıyla,( ) ikilileri bulunur. Yanıt 6 olur.

Örnek 133 a, b ve c ∈ R ve m ∈ Z+ olmak üzer,1

mn3 −an2−bn− c

ifadesi, her n ∈ Z için tamsayı olduguna göre, m sayısının alabilecegi kaç degervardır?Çözüm : Her ∈ Z için, 1

3 − 2 − − ifadesi tamsayı ise,

1

(+ 1)

3 − (+ 1)2 − (+ 1)−

ifadesi de tamsayıdır. Bu iki ifadenin farkından,3

2 +

µ3

− 2

¶+

1

− −

elde edilir.3

− 2 = ve

1

− − = denilirse, elde edilen

3

2 + + (∗)

Page 71: MATEMATİK OLİMPİYATLARINA HAZIRLIK 1Herbirkitaptaöncelikle,konuya veokonuileilgiliörnek ö˘gretici olabilecek sorulara yerverdim.Dahasonra,herbirkonuileilgilidünyada de˘gi¸

Temel Bilgiler 71

ifadesinin de, her ∈ Z için, tamsayı olması gerekir. Benzer sekilde,3

(+ 1)

2+ (+ 1) + (∗∗)

ifadesi de tamsayı olmalıdır. (∗∗) ve (∗) ifadelerinin farkından,6

+ +

3

elde edilir. Bu ifade de her ∈ Z için tamsayı olmalıdır. Benzer düsünceyle,µ6

(+ 1) + +

3

¶−µ6

+ +

3

¶=6

ifadesi de tamsayı olmalıdır. Buradan, pozitif tamsayısı 1 2 3 veya 6 olabilir.

Örnek 134 x2 = y2 +22

denklemini saglayan kaç (x, y, z) asal sayı üçlüsüvardır?Çözüm : Denklemi 2 − 2 = 2

2 biçiminde yazalım. Buradan,(− ) (+ ) = 2

2

olacaktır. (− ) ve (+ ) sayılarının biri tek ise digeri de tek, biri çift ise digeride çifttir. Sag taraf 2’nin bir kuvveti oldugundan her ikisi de 2’nin bir kuvvetine esitolması gerekir. O halde,

− = 2 ve + = 22−

diyelim. Bu denklemleri taraf tarafa toplarsak,2 = 2 + 2

2−

taraf tarafa çıkarırsak da2 = 2

2− − 2

elde edilir. ≥ 2 olması durumunda, ve sayıları çift sayı olacaklardır.

oldugundan ve ’den biri kesinlikle asal olmayacaktır. O halde, 2 olmalıdır. = 1 ise 2 = 21 + 22−1 ve 2 = 22−1 − 21 esitliklerinden,

= 1 + 22−2 ve = 22−2 − 1

bulunur. bir tek asal ise, sayısı 3’e bölünür. Bu durumda, ’in asal olması sadece = 3 iken mümkündür ki, − = 21 esitliginden, = 1 olur. Yani asal olmaz.O halde, bir çift asal, yani = 2 olmalıdır. Böylece, = 5 ve = 3 elde edilir.Sonuç olarak verilen denklem sisteminin tek ( ) asal sayı çözümü (5 3 2) olarakbulunur.